NUR 204 Med Surg Final Practice

Réussis tes devoirs et examens dès maintenant avec Quizwiz!

A client diagnosed with atherosclerosis has been prescribed Simvastatin (Zocor). Which of the following statement by the client indicates a need for further teaching?

"I only smoke 12 pack of cigarettes a day so I won't need to quit."

The nurse has taught a client with SCD how to prevent crises. The nurse knows the client needs further education after which statement?

"When I fly, I don't have to worry if the cab is pressurized or not."

What are the different pulse ratings?

-0 is absent. -1+ is diminished. -2+ is normal. -3+ is full pulse or slightly increased. -4+ is bounding.

A nurse assesses clients on a cardiac unit. Which clients should the nurse identify as at greatest risk for the development of acute pericarditis?

-A 36-year-old woman with systemic lupus erythematosus (SLE). -A 42-year-old man recovering from coronary artery bypass graft surgery. -An 80-year-old man with a bacterial infection of the respiratory tract.

A client with Type A+ blood requires PRBC infusion. Which of the following blood types can the patient receive?

-A+. -O+. -A-. -O-.

A nursing student studying acute coronary syndromes learns that the pain of a myocardial infarction (MI) differs from stable angina in what ways?

-Accompanied by SOB. -Feelings of fear or anxiety. -No relief from taking Nitro. -Pain occurs without known cause.

A nurse cares for a client with burn injuries during the resuscitation phase. Which actions are priorities during this phase?

-Administer analgesics. -Prevent wound infections. -Provide fluid replacement.

The nurse is caring for a client with suspected severe sepsis. What does the nurse prepare to do within 3 hours of the client being identified as being at risk?

-Administer antibiotics. -Draw serum lactate levels. -Obtain blood cultures.

A client is admitted to the hospital for sickle cell crisis and severe pain. Which of the following interventions would be appropriate for this client?

-Administer hypotonic solutions as ordered. -Obtain blood cultures if the patient develops a fever or other signs of infection. -Remove restrictive clothing. -Administer oxygen therapy. -Monitor for jaundice.

Unsealed Radiation?

-Administration via oral or IV route, or instillation into a specific body cavity 1. Source is not confined completely to one body area and it enters into body fluids and is excreted by such. 2. These excretions are harmful to others - precautions to not come into contact with fluids for healthcare workers and family/friends 3. Most of source is eliminated within 48 hours, then client/body fluids no longer radioactive

A nursing student planning to teach clients about risk factors for coronary artery disease (CAD) would include which topics?

-Advanced age. -Diabetes. -Ethnic background. -Smoking.

What are considered to be risk factors of HTN?

-African American ethnicity. -Smoking. -High Na+ diet. -Overweight. -High alcohol intake. -Stress. -Kidney disease. -Increasing age.

The nurse caring frequently for older adults in the hospital is aware of risk factors that place them at a higher risk for shock. For what factors would the nurse assess?

-Altered mobility/immobility. -Decreased thirst response. -Diminished immune response. -Malnutrition.

The student nurse studying shock understands that the common manifestations of this condition are directly related to which problems?

-Anaerobic metabolism. -Hypotension.

What overall client education should be provided for a client with a peripheral arterial problem?

-Antiplatelet. -Antihypertensives. -Antilipids.

A toddler with hemophilia fell onto their knees into rocks while running. Both knees have scrapes and bruises forming immediately. What should the parents do?

-Apply pressure for 15 minutes or until bleeding stop.s -Immobilize the knee if possible.

What nonpharmacologic comfort measures should the nurse include in the plan of care for a client with severe varicose veins?

-Applying elastic compression stockings. -Elevating the legs when sitting or lying. -Reminding the client to do leg exercises.

The nurse is preparing a client for a computed tomography (CT) scan, which requires infusion of radiopaque dye. Which of the following areas should the nurse address?

-Assess for seafood or iodine allergies. -Check the client's BUN and CRT levels. -Ensure metformin is not taken 48 hours before or after the procedure.

The nurse caring for hospitalized clients includes which actions on their care plans to reduce the possibility of the clients developing shock?

-Assessing and identifying clients at risk. -Performing proper hand hygiene. -Removing invasive lines as soon as possible. - Using aseptic technique during procedures.

A client is 1 day postoperative after a coronary artery bypass graft. What nonpharmacologic comfort measures does the nurse include when caring for this client?

-Assist the client into a position of comfort in bed. -Provide complimentary therapies such as music. -Remind the client to splint the incision when coughing.

Client is noted to have a low pitched murmur at the aortic valve. Which techniques should the nurse utilize to assess for this murmur?

-Auscultate with the bell of the stethoscope (hears low pitched sounds). -Auscultate at the 2nd ICS at the right sternal border.

A client with polycythemia vera has been taught ways to promote health and prolong life expectancy. (life expectancy is just 2 years if not treated/maintained.) Which would indicate the client has understood the teaching?

-Avoid tight or restrictive clothing. -Wear gloves outside when temp < 50. -Elevate your feet when sitting. -Stop activity and seek medical attention if chest pain develops. -Use an electric razor and soft bristled toothbrush while on anticoagulants. -Stop smoking.

What is the difference between benign and malignant tumors/growths?

-Benign growths are abnormal cellular growths but are not "cancerous." Ex: Skin tags, moles. -Malignant growths are cancerous. They are abnormal, serve no useful function, harms normal body tissues.

What factors affect blood pressure?

-Cardiac output. -Peripheral vascular resistance. -Volume in blood system. -Kidneys. -PaO2 and PaCO2. -Emotions.

What are the warning signs of cancer?

-Change in bowel or bladder habits -Any sore that does not heal -Unusual bleeding or discharge -Thickening or lump in breast or elsewhere -Indigestion or difficulty swallowing -Obvious change in wart or mole -Nagging cough or hoarseness

S/S of venous ulcers?

-Chronic and non healing. -Due to venous stasis. -Typically on ankles. -Wound bed pink with granulated tissue, superficial, and uneven edges.

A client has chronic PAD to their right upper extremity. The nurse would expect to find which S/S associated with this?

-Cool skin. -Drumstick appearance of the fingers. -Absence of hair on the fingers.

A client is being discharged on warfarin (Coumadin) therapy. What discharge instructions is the nurse required to provide?

-Dietary restrictions. -Followup laboratory monitoring. -Possible drug-drug interaction. -Reason to take medication.

A client with severe PAD is 2 hours post op from an Aorto fem-to-fem bypass grafting. The client reports pain in bilateral LE. The nurse notes the feet are red, warm, and bilateral pulses 2+. What is the nurse's most appropriate initial action?

-Document the findings.

Increased ICP signs and symptoms?

-Drop in LOC. -Drop in blood pressure and oxygen. -Patient reports headache. -Changes to pupils, causing blurred vision and photophobia. -Nausea and vomiting.

A nurse is preparing to administer Fresh Frozen Plasma. Which of the following should the nurse do for administration?

-Ensure consent is signed. -Administer immediately after thawing. -Ensure ABO compatibilty.

What conditions might the S1 sound be intensified?

-Exercise. -Hyperthyroidism. -Mitral stenosis.

What are generalized nursing interventions you can teach for good vascular health?

-Exercise. -No crossing of legs. -No tight clothing. -Do not sit for prolonged periods of time. -Drink plenty of fluids. -No caffeine or nicotine.

What lab findings would a nurse expect to find for a client with anemia?

-Hct 29%. -Hgb 8.5.

What clinical manifestations would the nurse expect to find in a patient with HTN?

-Headache. -Blurred vision. -Facial flushing. -Decreased urine output.

A nurse evaluates laboratory results for a client with heart failure. Which results should the nurse expect?

-Hematocrit 32.8%. -Serum Sodium 130 mEq/L. -Proteinuria. -Microalbuminuria.

What are interventions for acute arterial occlusion?

-Heparin infusion. -tPa. -Angiography. -Arteriotomy.

What are the most common risk factors for atherosclerosis?

-High LDL. -Low HDL. -Diabetes Mellitus. -Obesity. -Sedentary. -Smoking. -Stress. -Hypertension. -Genetics. -Age. -African American or Hispanic.

What are considered to be modifiable cardiovascular disease risk factors?

-High cholesterol. -Sedentary lifestyle. -Smoking. -Stress.

A nursing student learns about modifiable risk factors for coronary artery disease. Which factors does this include?

-Hypertension. -Obesity. -Smoking. -Stress.

After teaching a client with congestive heart failure (CHF), the nurse assesses the clients understanding. Which client statements indicate a correct understanding of the teaching related to nutritional intake?

-Ill read the nutritional labels on food items for salt content. -I will eat oatmeal for breakfast instead of ham and eggs. - Substituting fresh vegetables for canned ones will lower my salt intake.

What are considered to be common risk factors of cancer in general ?

-Industrial chemicals. -Tobacco. -Radiation. -Viruses. -Obesity. -Low fiber, high preservative, hight animal fat, and high red meat diet. -Increasing age. -Genetics. -Immunosuppressed.

Physical changes in cancer?

-Low WBC, RBC, and platelets. -Altered GI function. Altered peripheral nerves. -Motor and sensory changes. -Pain. -Altered respiratory and cardiac function.

A client with severe PAD has just returned from having a left fem-pop bypass procedure. The nurse will complete which of the following in a focused assessment for this surgery?

-Mark the distal pulses of the left foot. -Monitor vital signs for hyper or hypotension. -Maintain the heparin drip per protocol. -Monitor incision sites for hematoma and bleeding.

S/S of arterial ulcers?

-More acute. -Due to no arterial flow. -Typically on toes. -Wound bed pale, little to no granulation tissue, eschar, and even edges.

A nurse assesses a client who is diagnosed with infective endocarditis. Which assessment findings should the nurse expect?

-Night sweats. -Cardiac murmur. -Abdominal bloating.

What are non modifiable risk factors for cardiovascular disease?

-Older age. -Males. -African and Hispanic. -Family history or genetics. -CAD or Valve disease. -Recurrent tonsillitis, strep infections, or rheumatic fever.

What are the 6 P's of ischemia?

-Pain. -Pallor. -Pulselessness. -Paresthesia. -Paralysis. -Poikilothermy.

S/S of anemia include which of the following?

-Pallor of ears, nail bed, conjunctivae, and around mouth. -Tachycardia. -Dyspnea on exertion. -Low O2 levels.

Potential causes of a sickle cell crisis include which of the following?

-Poor fluid intake. -Excessive intake of caffeinated beverages. -Extreme, prolonged exercise. -Hot climate or temperature. -Trauma.

What are specific nursing interventions you can teach for good ARTERIAL health?

-Positioning. -Avoid the cold. -Avoid direct heat on site. -Cholesterol medicaations. -Antiplatelets. -Foot care.

What are specific nursing interventions you can teach for good VENOUS health?

-Positioning. -SCD's.

What are "late symptoms" for increased ICP?

-Projectile vomiting. -Paresis/Paralysis. -Cheyne stokes. -Unresponsive pupils. -Decerebrate and Decorticate posturing. -Cushing's triad.

A nurse plans care for a client with burn injuries. Which interventions should the nurse include in this clients plan of care to ensure adequatenutrition?

-Provide at least 5000 kcal/day. -Administer a diet high in protein. -Collaborate with a registered dietitian. -Offer frequent high-calorie snacks.

A nurse is assessing a client with left-sided heart failure. For which clinical manifestations should the nurse assess?

-Pulmonary crackles. -Confusion, restlessness. -Cough that worsens at night.

The nurse is auscultating the apical heart beat for a full minute. The nurse is listening for what items during this time?

-Rate. -Rhythm. -Length of cardiac cycle (systole and diastole). -Valve function. -Friction rub.

How do we manage atherosclerosis?

-Risk factor modification. -Exercise. -Low fat, low cholesterol, and increased fiber diet. -Medications for cholesterol (Statins).

Nonsurgical management of aneurysms?

-Risk factor modification. -Monitor growth. -Prevent HTN.

What are some oncological emergencies?

-Sepsis. -DIC. -SIADH. -Spinal cord compression. -Hypercalcemia. -Superior vena cava syndrome. -Tumor lysis syndrome.

What is Cushing's triad?

-Severe hypertension. -Widened pulse pressure. -Bradycardia.

A nurse cares for an older client with burn injuries. Which age-related changes are paired appropriately with their complications from the burn injuries?

-Slower healing time Increased risk for loss of function from contracture formation. -Reduced thoracic compliance Increased risk for atelectasis. -High incidence of cardiac impairments Increased risk for acute kidney injury.

Treatments for cancer?

-Surgery. -Teletherapy. -Brachytherapy, either sealed or unsealed. -Chemo.

A client with severe anemia requires a blood transfusion. The nurse will complete which of the following during the administration process.

-Verify a patent IV, at least 20 gauge, is available. -Check vital signs before, at the beginning of, periodically throughout, and after administration of blood. -Utilize filtered blood tubing Y'd in with NS to administer the blood. -Infuse the blood at prescribed rate while ensuring it is completed within 4 hours of being removed from blood bank. -Monitor for s/s of fluid volume overload.

What is metastasis?

-When cancer cells move from original location to other sites via local seeding, though the blood (most common), or lymphatic system. -Most common are bone, lung, brain, & liver.

Nausea & Vomiting?

1. Administer antiemetics before, during and after chemo 2. Avoid odors 3. Cool, quiet environment 4. Encourage hydration - IV Fluids giving before and during chemo; promote fluid intake of at least 2000mL/day

Alopecia?

1. Buy a wig BEFORE beginning treatment 2. Cut hair short 3. Educate that hair growth should restart several months after last treatment Chemo cold pack to prevent alopecia

Mucositis(sores in mucous membranes)?

1. Frequently assess mouth and perform oral hygiene 2. Teach patient to perform good and frequent oral hygiene(soft bristle tooth brush, gentle flossing daily, rinse mouth with plain water or saline every hour while awake, avoid mouth washes that contain alcohol or glycerin) Avoid citric/acidic drinks/foods 3. Administer rinse compounds to help with pain and discomfort especially after meals. 4. Moisturize mouth/lips 5. Hydration 6. Oral ice packs

For both N/V and mucositis + Loss of appetite/taste changes, and Diarrhea?

1. High calorie diet with protein supplement 2. Small, frequent meals 3. Meals pt prefers Avoid spicy foods; warm foods; caffeine, high fiber foods, or other GI stimulants

Anemia(decreased RBC)?

1. Monitor CBC. 2. Administer growth factors to stimulate marrow production(epoetin alfa) if appropriate (not for Leukemia) 3. Monitor for fatigue and weakness. 4. Assist with ADL's as necessary. Prioritize activities; rest as needed 5. Small frequent meals high carb/protein; supplements; dietician 6. Transfuse PRBCs prn

Neutropenia(decreased WBC)-leading to immunosuppression and risk for infection?

1. Monitor for infection- VS Q 4hours(temp >100); (Temp increase of 1 degree for severely depressed immune system); WBC; and segmented neutrophil count (segmented are "mature" neutrophils & bands are immature neutrophils); when segs are below 18% OR ANC is <500, initiate neutropenic precautions) ; neutrophils are vital in fighting infection - they are one of the first responders - ingest invaders and then release enzymes to kill them 2. Assess skin, mucous membranes, drainage from wound (if appropriate) & lung sounds. Monitor for other S/S infection - ex: dysuria, malodorous smells, etc 2. Good hand washing 3. Aseptic technique for invasive procedures; prevent skin breakdown; good foley care; IS/CDB 4. Check with HCP before receiving vaccine - live vaccines not able to receive (small pox, rotavirus, shingles) 5. If infection suspected, obtain cultures and blood work; then hang antimicrobials as indicated ***Review "REVERSE" or "Neutropenic" isolation precautions*** Private room; No raw fruit/veggies, no fresh flowers/potted plants, no standing water; no sick visitors, gown/gloves/mask, avoid procedures that could introduce infection if possible (foley, enema, etc), visitors wear PPE too, restrict ill visitors ***Review COMMUNITY ed (not all in hospital) - Avoid crowds, those sick, wear mask if around those; Don't share personal items; avoid salad bars, raw fruit/veggie, undercooked meat, eggs, etc; no fresh plants/flowers; run tooth brush through dishwasher weekly or use bleach rinse; avoid turtle/reptile pets; etc

Fertility?

1. Need for contraception as most drugs have teratogenic effects 2. Discuss infertility may be irreversible Sperm or egg banking

Administering Chemo?

1. Well vented area 2. Wear PPE - gloves, gown, goggles, mask while preparing, administering, and for 48 hours after admin if handling bodily fliud/excrement (goggles, mask, double gloves, gown) 3. PREGNANT women CAN NOT prepare/admin chemo 4. Discard IV tubing/bag in biohazard containers 5. Follow protocol/notify biohazard team if spill occurs 6. Monitor for phlebitis or extravasation (leakage into surrounding tissues) - leave IV in place as antidote may be admin; heat or ice depending on med; NOTIFY HCP 7. Do NOT crush, split, or chew oral chemo

The nurse is making initial rounds at the beginning of the shift and notes that the TPN bag of an assigned patient is empty. Which solution should the nurse hang until another bag of TPN is mixed and delivered to the nursing unit?

10% Dextrose in water.

A nurse uses the rule of nines to assess a client with burn injuries to the entire back region and left arm. How should the nurse document the percentage of the clients body that sustained burns?

27%.

The nurse instructs a client on the steps needed to obtain a peak expiratory flow rate. In which order should these steps occur? 1. Take as deep a breath as possible. 2. Stand up (unless you have a physical disability). 3. Place the meter in your mouth, and close your lips around the mouthpiece. 4. Make sure the device reads zero or is at base level. 5. Blow out as hard and as fast as possible for 1 to 2 seconds. 6. Write down the value obtained. 7. Repeat the process two additional times, and record the highest number in your chart.

4, 2, 1, 3, 5, 6, 7.

he nurse instructs a client on how to correctly use an inhaler with a spacer. In which order should these steps occur? 1. Press down firmly on the canister to release one dose of medication. 2. Breathe in slowly and deeply. 3. Shake the whole unit vigorously three or four times. 4. Insert the mouthpiece of the inhaler into the nonmouthpiece end of the spacer. 5. Place the mouthpiece into your mouth, over the tongue, and seal your lips tightly around the mouthpiece. 6. Remove the mouthpiece from your mouth, keep your lips closed, and hold your breath for at least 10 seconds.

4, 3, 5, 1, 2, 6.

Sealed Radiation?

A 1. sealed, temporary or permanent radiation source (solid implant) is placed inside pt 2. Client emits radiation while implant is in place, but body fluids are NOT radioactive 3. May stay in place minutes (high dose) to days/weeks, or may be permanent (low dose and stops giving off after a few weeks/months) 4. Client is no longer radioactive once implant is removed or after several weeks/months if permanent 5. Nursing Implications for when radioactive: a. Pt to be in private room with private bath b. Radiation caution sign posted on door; keep door closed when possible c. Organize/cluster activities to minimize exposure to pt d. Rotate nurses/workers to minimize exposure e. Wear a dosimeter film badge to measure exposure f. Lead shielding (apron) may be worn to reduce exposure; don't expose "back" to pt g. Never care for more than 1 client with sealed implant at a time h. Do not allow PREGNANT individuals to care for pt i. Do not allow children <16yrs or pregnant women to visit pt j. Limit visitors to 30 minutes per day; stay at least 6 feet from source k. Save bed linens/dressings until source is removed; then dispose of linens in usual manner (or per facility policy) l. Other equipment can be removed at any time m. If the implant dislodges: i. Encourage pt to lie still ii. Use a long handled forceps to retrieve the implant iii. Deposit it into a lead container; follow facility policy for how to further remove/handle situation iv. Contact the oncologist v. Document event and actions taken

A nurse assesses clients on a cardiac unit. Which client should the nurse identify as being at greatest risk for the development of left-sided heart failure?

A 36-year-old woman with aortic stenosis.

A nurse is reinforcing teaching with a group of clients about transient ischemic attacks (TIAs). Which of the following information should the nurse include in the teaching?

A TIA can precede an ischemic stroke.

The nurse is evaluating the status of a client who had a craniotomy 3 days ago. The nurse would suspect the client is developing meningitis as a complication of surgery if the client exhibits?

A positive Brudzinski's sign.

Vitamin B12 deficiency anemia leads to macrocytic (large cell) anemia. Causes of Vitamin B12 deficiency include which of the following?

A vegan diet.

A client in sickle cell crisis is dehydrated and in the emergency department. The nurse plans to start an IV. Which fluid choice is best? A) 0.45% normal saline B) 0.9% normal saline C) Dextrose 50% (D50) D) Lactated Ringers solution

A, 0.45% normal saline.

A nurse is providing pneumonia vaccinations in a community setting. Due to limited finances, the event organizers must limit giving the vaccination to priority groups. What clients would be considered a priority when administering the pneumoniavaccination? (Select all that apply.) A) 22-year-old client with asthma B) Client who had a cholecystectomy last year C) Client with well-controlled diabetes D) Healthy 72-year-old client E) Client who is taking medication for hypertension

A, 22-year-old client with asthma. C, Client with well-controlled diabetes. D, Healthy 73-year-old client. E, Client who is taking medication for hypertension.

A nurse triages clients arriving at the hospital after a mass casualty. Which clients are correctly classified? (Select all that apply.) A) A 35-year-old female with severe chest pain: red tag. B) A 42-year-old male with full-thickness body burns: green tag. C) A 55-year-old female with a scalp laceration: black tag. D) A 60-year-old male with an open fracture with distal pulses: yellow tag. E) An 88-year-old male with shortness of breath and chest bruises: green tag

A, A 35-year-old female with severe chest pain: red tag. D, A 60-year-old male with an open fracture with distal pulses: yellow tag.

A nurse assesses clients on a cardiac unit. Which client should the nurse identify as being at greatest risk for the development of left-sided heart failure? A) A 36-year-old woman with aortic stenosis B) A 42-year-old man with pulmonary hypertension C) A 59-year-old woman who smokes cigarettes daily D) A 70-year-old man who had a cerebral vascular accident

A, A 36-year-old woman with aortic stenosis.

A nurse assesses clients on a cardiac unit. Which clients should the nurse identify as at greatest risk for the development of acute pericarditis? (Select all that apply.) A) A 36-year-old woman with systemic lupus erythematosus (SLE) B) A 42-year-old man recovering from coronary artery bypass graft surgery C) A 59-year-old woman recovering from a hysterectomy D) An 80-year-old man with a bacterial infection of the respiratory tract E) An 88-year-old woman with a stage III sacral ulcer

A, A 36-year-old woman with systemic lupus erythematosus (SLE). B, A 42-year-old man recovering from coronary artery bypass graft surgery. D, An 80-year-old man with a bacterial infection of the respiratory tract.

A nurse is providing community education on the seven warning signs of cancer. Which signs are included? (Select all that apply.) A) A sore that does not heal. B) Changes in menstrual patterns. C) Indigestion or trouble swallowing. D) Near-daily abdominal pain. E) Obvious change in a mole.

A, A sore that does not heal. B, Changes in menstrual patterns. C, Indigestion or trouble swallowing. E, Obvious change in a mole.

A client with a new pulmonary embolism (PE) is anxious. What nursing actions are most appropriate? (Select all that apply.) A) Acknowledge the frightening nature of the illness. B) Delegate a back rub to the unlicensed assistive personnel (UAP). C) Give simple explanations of what is happening. D) Request a prescription for antianxiety medication. E) Stay with the client and speak in a quiet, calm voice.

A, Acknowledge the frightening nature of the illness. B, Delegate a back rub to the unlicensed assistive personnel (UAP). C, Give simple explanations of what is happening. E, Stay with the client and speak in a quiet, calm voice.

The nurse caring for mechanically ventilated clients uses best practices to prevent ventilator-associated pneumonia. What actions are included in this practice? (Select all that apply.) A) Adherence to proper hand hygiene B) Administering anti-ulcer medication C) Elevating the head of the bed D) Providing oral care per protocol E) Suctioning the client on a regular schedule

A, Adherence to proper hand hygiene B, Administering anti-ulcer medication C, Elevating the head of the bed D, Providing oral care per protocol.

A provider prescribes a rewarming bath for a client who presents with partial-thickness frostbite. Which action should the nurse take prior to starting this treatment? A) Administer intravenous morphine. B) Wrap the limb with a compression dressing. C) Massage the frostbitten areas. D) Assess the limb for compartment syndrome.

A, Administer IV morphine.

A nurse cares for a client with burn injuries during the resuscitation phase. Which actions are priorities during this phase? (Select all that apply.) A) Administer analgesics. B) Prevent wound infections. C) Provide fluid replacement. D) Decrease core temperature. E) Initiate physical therapy.

A, Administer analgesics. B, Prevent wound infections. C, Provide fluid replacement.

An emergency department nurse cares for a middle-aged mountain climber who is confused and exhibits bizarre behaviors. After administering oxygen, which priority intervention should the nurse implement? A) Administer dexamethasone (Decadron). B) Complete a minimental state examination. C) Prepare the client for computed tomography of the brain. D) Request a psychiatric consult.

A, Administer dexamthasone.

An emergency department nurse plans care for a client who is admitted with heat stroke. Which interventions should the nurse include in this clients plan of care? (Select all that apply.) A) Administer oxygen via mask or nasal cannula. B) Administer ibuprofen, an antipyretic medication. C) Apply cooling techniques until core body temperature is less than 101 F. D) Infuse 0.9% sodium chloride via a large-bore intravenous cannula. E) Obtain baseline serum electrolytes and cardiac enzymes.

A, Administer oxygen via mask or nasal canula. D, Infuse 0.9% sodium chloride bia a large-bore IV cannula. E, Obtain baseline serum electrolytes and cardiac enzymes.

A client presents to the emergency department in sickle cell crisis. What intervention by the nurse takes priority? A) Administer oxygen. B) Apply an oximetry probe. C) Give pain medication. D) Start an IV line.

A, Administer oxygen.

A nurse cares for a client with burn injuries. Which intervention should the nurse implement to appropriately reduce the clients pain? A) Administer the prescribed intravenous morphine sulfate. B) Apply ice to skin around the burn wound for 20 minutes. C) Administer prescribed intramuscular ketorolac (Toradol). D) Decrease tactile stimulation near the burn injuries.

A, Administer the prescribed IV morphine sulfate.

A nursing student planning to teach clients about risk factors for coronary artery disease (CAD) would include which topics? (Select all that apply.) A) Advanced age B) Diabetes C) Ethnic background D) Medication use E) Smoking

A, Advanced age B, Diabetes C, Ethnic background E, Smoking

A client is receiving norepinephrine (Levophed) for shock. What assessment finding best indicates a therapeutic effect from this drug? A) Alert and oriented, answering questions B) Client denial of chest pain or chest pressure C) IV site without redness or swelling D) Urine output of 30 mL/hr for 2 hours

A, Alert and oriented, answering questions.

A nurse is caring for a client who is on mechanical ventilation. What actions will promote comfort in this client? (Select all that apply.) A) Allow visitors at the clients bedside. B) Ensure the client can communicate if awake. C) Keep the television tuned to a favorite channel. D) Provide back and hand massages when turning. E) Turn the client every 2 hours or more.

A, Allow visitors at the clients bedside. B, Ensure the client can communicate if awake. D, Provide back and hand massages when turning. E, Turn the client every 2 hours or more.

A nurse works on an oncology unit and delegates personal hygiene to an unlicensed assistive personnel (UAP). What action by the UAP requires intervention from the nurse? A) Allowing a very tired client to skip oral hygiene and sleep. B) Assisting clients with washing the perianal area every 12 hours C) Helping the client use a soft-bristled toothbrush for oral care D) Reminding the client to rinse the mouth with water or saline.

A, Allowing a very tired client to skip oral hygiene and sleep.

A client in the emergency department has several broken ribs. What care measure will best promote comfort? A) Allowing the client to choose the position in bed B) Humidifying the supplemental oxygen C) Offering frequent, small drinks of water D) Providing warmed blankets

A, Allowing the client to choose the position in bed.

The nurse caring frequently for older adults in the hospital is aware of risk factors that place them at a higher risk for shock. For what factors would the nurse assess? (Select all that apply.) A) Altered mobility/immobility B) Decreased thirst response C) Diminished immune response D) Malnutrition E) Overhydration

A, Altered mobility/immobility B, Decreased thirst response C, Diminished immune response D, Malnutrition

The student nurse studying shock understands that the common manifestations of this condition are directly related to which problems? (Select all that apply.) A) Anaerobic metabolism B) Hyperglycemia C) Hypotension D) Impaired renal perfusion E) Increased perfusion

A, Anaerobic metabolism. C, Hypotension.

The emergency department (ED) manager is reviewing client charts to determine how well the staff performs when treating clients with community-acquired pneumonia. What outcome demonstrates that goals for this client type have been met? A) Antibiotics started before admission B) Blood cultures obtained within 20 minutes C) Chest x-ray obtained within 30 minutes D) Pulse oximetry obtained on all clients

A, Antibiotics started before admission.

An emergency room nurse assesses a client who was rescued from a home fire. The client suddenly develops a loud, brassy cough. Which action should the nurse take first? A) Apply oxygen and continuous pulse oximetry. B) Provide small quantities of ice chips and sips of water. C) Request a prescription for an antitussive medication. D) Ask the respiratory therapist to provide humidified air.

A, Apply oxygen and continuous pulse oximetry.

A client has been brought to the emergency department after being shot multiple times. What action should the nurse perform first? A) Apply personal protective equipment. B) Notify local law enforcement officials. C) Obtain universal donor blood. D) Prepare the client for emergency surgery.

A, Apply personal protective equipment.

A nurse is working with a client who takes atorvastatin (Lipitor). The clients recent laboratory results include a blood urea nitrogen (BUN) of 33 mg/dL and creatinine of 2.8 mg/dL. What action by the nurse is best? A) Ask if the client eats grapefruit. B) Assess the client for dehydration. C) Facilitate admission to the hospital. D) Obtain a random urinalysis.

A, Ask if the client eats grapefruit.

A nurse plans care for a client who has chronic obstructive pulmonary disease and thick, tenacious secretions. Which interventions should the nurse include in this clients plan of care? (Select all that apply.) A) Ask the client to drink 2 liters of fluids daily. B) Add humidity to the prescribed oxygen. C) Suction the client every 2 to 3 hours. D) Use a vibrating positive expiratory pressure device. E) Encourage diaphragmatic breathing.

A, Ask the client to drink 2 liters of fluids daily. B, Add humidity to prescribed oxygen. D, Use a vibrating positive expiratory pressure device.

A nurse cares for a client after radiation therapy for lung cancer. The client reports a sore throat. Which action should the nurse take first? A) Ask the client to gargle with mouthwash containing lidocaine. B) Administer prescribed intravenous pain medications. C) Explain that soreness is normal and will improve in a couple days. D) Assess the clients neck for redness and swelling.

A, Ask the client to gargle with mouthwash containing lidocaine.

A client has been hospitalized with tuberculosis (TB). The clients spouse is fearful of entering the room where the client is in isolation and refuses to visit. What action by the nurse is best? A) Ask the spouse to explain the fear of visiting in further detail. B) Inform the spouse the precautions are meant to keep other clients safe. C) Show the spouse how to follow the isolation precautions to avoid illness. D) Tell the spouse that he or she has already been exposed, so its safe to visit.

A, Ask the spouse to explain the fear of visiting in further detail.

A client receiving chemotherapy has a white blood cell count of 1000/mm3. What actions by the nurse are most appropriate? (Select all that apply.) A) Assess all mucous membranes every 4 to 8 hours. B) Do not allow the client to eat meat or poultry. C) Listen to lung sounds and monitor for cough. D) Monitor the venous access device appearance with vital signs. E) Take and record vital signs every 4 to 8 hours.

A, Assess all mucous membranes every 4-8 hours. C, Listen to lung sounds and monitor for cough. D, Monitor the venous access device appearance with vital signs. E, Take and record vital signs every 4-8 hours.

A nurse is caring for a client with a history of renal insufficiency who is scheduled for a cardiac catheterization. Which actions should the nurse take prior to the catheterization? (Select all that apply.) A) Assess for allergies to iodine. B) Administer intravenous fluids. C) Assess blood urea nitrogen (BUN) and creatinine results. D) Insert a Foley catheter. E) Administer a prophylactic antibiotic. F) Insert a central venous catheter.

A, Assess for allergies to iodine. B, Administer intravenous fluids. C, Assess blood urea nitrogen (BUN) and creatinine results.

A client appears dyspneic, but the oxygen saturation is 97%. What action by the nurse is best? A) Assess for other manifestations of hypoxia. B) Change the sensor on the pulse oximeter. C) Obtain a new oximeter from central supply. D) Tell the client to take slow, deep breaths.

A, Assess for other manifestations of hypoxia.

While assessing a client on a cardiac unit, a nurse identifies the presence of an S3 gallop. Which action should the nurse take next? A) Assess for symptoms of left-sided heart failure. B) Document this as a normal finding. C) Call the health care provider immediately. D) Transfer the client to the intensive care unit.

A, Assess for symptoms of left-sided heart failure.

A nurse is caring for a client on mechanical ventilation and finds the client agitated and thrashing about. What action by the nurse is most appropriate? A) Assess the cause of the agitation. B) Reassure the client that he or she is safe. C) Restrain the clients hands. D) Sedate the client immediately.

A, Assess the cause of the agitation.

A nurse is caring for a client on IV infusion of heparin. What actions does this nurse include in the clients plan of care? (Select all that apply.) A) Assess the client for bleeding. B) Monitor the daily activated partial thromboplastin time (aPTT) results. C) Stop the IV for aPTT above baseline. D) Use an IV pump for the infusion. E) Weigh the client daily on the same scale.

A, Assess the client for bleeding. B, Monitor the daily activated partial thromboplastin time (aPTT) results. D, Use an IV pump for the infusion.

A client with a history of prostate cancer is in the clinic and reports new onset of severe low back pain. What action by the nurse is most important? A) Assess the clients gait and balance. B) Ask the client about the ease of urine flow. C) Document the report completely. D) Inquire about the clients job risks.

A, Assess the clients gait and balance.

A client with a history of heart failure and hypertension is in the clinic for a follow-up visit. The client is on lisinopril (Prinivil) and warfarin (Coumadin). The client reports new- onset cough. What action by the nurse is most appropriate? A) Assess the clients lung sounds and oxygenation. B) Instruct the client on another antihypertensive. C) Obtain a set of vital signs and document them. D) Remind the client that cough is a side effect of Prinivil.

A, Assess the clients lung sounds and oxygenation.

An older client is hospitalized with Guillain-Barr syndrome. A family member tells the nurse the client is restless and seems confused. What action by the nurse is best? A) Assess the clients oxygen saturation. B) Check the medication list for interactions. C) Place the client on a bed alarm. D) Put the client on safety precautions.

A, Assess the clients oxygen saturation.

A nurse admits a client who is experiencing an exacerbation of heart failure. Which action should the nurse take first? A) Assess the clients respiratory status. B) Draw blood to assess the clients serum electrolytes. C) Administer intravenous furosemide (Lasix). D) Ask the client about current medications.

A, Assess the clients respiratory status.

A nurse is assessing a dark-skinned client for pallor. What action is best? A) Assess the conjunctiva of the eye. B) Have the client open the hand widely. C) Look at the roof of the clients mouth. D) Palpate for areas of mild swelling.

A, Assess the conjunctiva of the eye.

A client has been diagnosed with a deep vein thrombosis and is to be discharged on warfarin (Coumadin). The client is adamant about refusing the drug because its dangerous. What action by the nurse is best? A) Assess the reason behind the clients fear. B) Remind the client about laboratory monitoring. C) Tell the client drugs are safer today than before. D) Warn the client about consequences of noncompliance.

A, Assess the reason behind the clients fear.

The nurse caring for hospitalized clients includes which actions on their care plans to reduce the possibility of the clients developing shock? (Select all that apply.) A) Assessing and identifying clients at risk B) Monitoring the daily white blood cell count C) Performing proper hand hygiene D) Removing invasive lines as soon as possible E) Using aseptic technique during procedures

A, Assessing and identifying clients at risk. C, Performing proper hand hygiene. D, Removing invasive lines as soon as possible. E, Using aseptic technique during procedures.

A client is receiving chemotherapy through a peripheral IV line. What action by the nurse is most important? A) Assessing the IV site every hour. B) Educating the client on side effects. C) Monitoring the client for nausea. D) Providing warm packs for comfort

A, Assessing the IV site every hour.

A client has mucositis. What actions by the nurse will improve the clients nutrition? (Select all that apply.) A) Assist with rinsing the mouth with saline frequently. B) Encourage the client to eat room-temperature foods. C) Give the client hot liquids to hold in the mouth. D) Provide local anesthetic medications to swish and spit. E) Remind the client to brush teeth gently after each meal.

A, Assist with rinsing the mouth with saline frequently. B, Encourage the client to eat room-temperature foods. D, Provide local anesthetic medications to swish and spit. E, Remind the client to brush teeth gently after each meal.

A client has been diagnosed with an empyema. What interventions should the nurse anticipate providing to this client? (Select all that apply.) A) Assisting with chest tube insertion B) Facilitating pleural fluid sampling C) Performing frequent respiratory assessment D) Providing antipyretics as needed E) Suctioning deeply every 4 hours

A, Assisting with chest tube insertion. B, Facilitating pleural fluid sampling. C, Performing frequent respiratory assessment. D, Providing antipyretics as needed.

A nurse teaches a client who has chronic obstructive pulmonary disease. Which statements related to nutrition should the nurse include in this clients teaching? (Select all that apply.) A) Avoid drinking fluids just before and during meals. B) Rest before meals if you have dyspnea. C) Have about six small meals a day. D) Eat high-fiber foods to promote gastric emptying. E) Increase carbohydrate intake for energy.

A, Avoid drinking fluids just before and during meals. B, Rest before meals if you have dyspnea. C, Have about six small meals a day.

A client is taking long-term corticosteroids for myasthenia gravis. What teaching is most important? A) Avoid large crowds and people who are ill. B) Check blood sugars four times a day. C) Use two forms of contraception. D) Wear properly fitting socks and shoes.

A, Avoid large crowds and people who are ill.

A nurse is teaching a client with heart failure who has been prescribed enalapril (Vasotec). Which statement should the nurse include in this clients teaching? A) Avoid using salt substitutes. B) Take your medication with food. C) Avoid using aspirin-containing products. D) Check your pulse daily.

A, Avoid using salt substitutes.

The nurse caring for oncology clients knows that which form of metastasis is the most common? A) Bloodborne. B) Direct invasion. C) Lymphatic spread. D) Via bone marrow.

A, Bloodborne.

A client admitted for pneumonia has been tachypneic for several days. When the nurse starts an IV to give fluids, the client questions this action, saying I have been drinking tons of water. How am I dehydrated? What response by the nurse is best? A) Breathing so quickly can be dehydrating. B) Everyone with pneumonia is dehydrated. C) This is really just to administer your antibiotics. D) Why do you think you are so dehydrated?

A, Breathing so quickly can be dehydrating.

A client is in the early stages of shock and is restless. What comfort measures does the nurse delegate to the nursing student? (Select all that apply.) A) Bringing the client warm blankets B) Giving the client hot tea to drink C) Massaging the clients painful legs D) Reorienting the client as needed E) Sitting with the client for reassurance

A, Bringing the client warm blankets. D, Reorienting the client as needed. E, Sitting with the client for reassurance.

A client has a traumatic brain injury. The nurse assesses the following: pulse change from 82 to 60 beats/min, pulse pressure increase from 26 to 40 mm Hg, and respiratory irregularities. What action by the nurse takes priority? A) Call the provider or Rapid Response Team. B) Increase the rate of the IV fluid administration. C) Notify respiratory therapy for a breathing treatment. D) Prepare to give IV pain medication.

A, Call the provider or Rapid Response Team.

A client has a platelet count of 9000/mm3. The nurse finds the client confused and mumbling. What action takes priority? A) Calling the Rapid Response Team B) Delegating taking a set of vital signs C) Instituting bleeding precautions D) Placing the client on bedrest

A, Calling the Rapid Response Team.

A nurse is preparing to administer IV chemotherapy. What supplies does this nurse need? (Select all that apply.) A) Chemo gloves. B) Facemask. C) Isolation gown. D) N95 respirator. E) Shoe covers

A, Chemo gloves. B, Face mask. C, Isolation gown.

A nurse is caring for four clients with leukemia. After hand-off report, which client should the nurse see first? A) Client who had two bloody diarrhea stools this morning B) Client who has been premedicated for nausea prior to chemotherapy C) Client with a respiratory rate change from 18 to 22 breaths/min D) Client with an unchanged lesion to the lower right lateral malleolus

A, Client who had two bloody diarrhea stools this morning.

The nurse gets the hand-off report on four clients. Which client should the nurse assess first? A) Client with a blood pressure change of 128/74 to 110/88 mm Hg B) Client with oxygen saturation unchanged at 94% C) Client with a pulse change of 100 to 88 beats/min D) Client with urine output of 40 mL/hr for the last 2 hours

A, Client with a blood pressure change of 128/74 to 110/88 mm Hg.

he health care provider tells the nurse that a client is to be started on a platelet inhibitor. About what drug does the nurse plan to teach the client? A) Clopidogrel (Plavix) B) Enoxaparin (Lovenox) C) Reteplase (Retavase) D) Warfarin (Coumadin)

A, Clopidogrel (Plavix).

A nurse assesses a client who has a nasal fracture. The client reports constant nasal drainage, a headache, and difficulty with vision. Which action should the nurse take next? A) Collect the nasal drainage on a piece of filter paper. B) Encourage the client to blow his or her nose. C) Perform a test focused on a neurologic examination. D) Palpate the nose, face, and neck.

A, Collect the nasal drainage on a piece of filter paper.

While assessing a client who has facial trauma, the nurse auscultates stridor. The client is anxious and restless. Which action should the nurse take first? A) Contact the provider and prepare for intubation. B) Administer prescribed albuterol nebulizer therapy. C) Place the client in high-Fowlers position. D) Ask the client to perform deep-breathing exercises.

A, Contact the provider and prepare for intubation.

A nurse is assessing a client with peripheral artery disease (PAD). The client states walking five blocks is possible without pain. What question asked next by the nurse will give the best information? A) Could you walk further than that a few months ago? B) Do you walk mostly uphill, downhill, or on flat surfaces? C) Have you ever considered swimming instead of walking? D) How much pain medication do you take each day?

A, Could you walk further than that a few months ago?

A nurse administers topical gentamicin sulfate (Garamycin) to a clients burn injury. Which laboratory value should the nurse monitor while the client is prescribed this therapy? A) Creatinine B) Red blood cells C) Sodium D) Magnesium

A, Creatinine.

A nurse caring for a client with sickle cell disease (SCD) reviews the clients laboratory work. Which finding should the nurse report to the provider? A) Creatinine: 2.9 mg/dL B) Hematocrit: 30% C) Sodium: 147 mEq/L D) White blood cell count: 12,000/mm3

A, Creatinine: 2.9 mg/dL.

A nurse working with clients with sickle cell disease (SCD) teaches about self- management to prevent exacerbations and sickle cell crises. What factors should clients be taught to avoid? (Select all that apply.) A) Dehydration B) Exercise C) Extreme stress D) High altitudes E) Pregnancy

A, Dehydration. C, Extreme stress. D, High altitudes. E, Pregnancy.

A client is being discharged on warfarin (Coumadin) therapy. What discharge instructions is the nurse required to provide? (Select all that apply.) A) Dietary restrictions B) Driving restrictions C) Follow-up laboratory monitoring D) Possible drug-drug interactions E) Reason to take medication

A, Dietary restrictions. C, Follow-up laboratory monitoring D, Possible drug-drug interactions. E, Reason to take medication.

A client is in the clinic a month after having a myocardial infarction. The client reports sleeping well since moving into the guest bedroom. What response by the nurse is best? A) Do you have any concerns about sexuality? B) Im glad to hear you are sleeping well now. C) Sleep near your spouse in case of emergency. D) Why would you move into the guest room?

A, Do you have any concerns about sexuality?

A nurse obtains a focused health history for a client who is suspected of having bacterial meningitis. Which question should the nurse ask? A) Do you live in a crowded residence? B) When was your last tetanus vaccination? C) Have you had any viral infections recently? D) Have you traveled out of the country in the last month?

A, Do you live in a crowded residence?

A nurse auscultates a harsh hollow sound over a clients trachea and larynx. Which action should the nurse take first? A) Document the findings. B) Administer oxygen therapy. C) Position the client in high-Fowlers position. D) Administer prescribed albuterol.

A, Document the findings.

The nurse is caring for a client with leukemia who has the priority problem of fatigue. What action by the client best indicates that an important goal for this problem has been met? A) Doing activities of daily living (ADLs) using rest periods B) Helping plan a daily activity schedule C) Requesting a sleeping pill at night D) Telling visitors to leave when fatigued

A, Doing activities of daily living (ADLs) using rest periods.

A client has been diagnosed with tuberculosis (TB). What action by the nurse takes highest priority? A) Educating the client on adherence to the treatment regimen B) Encouraging the client to eat a well-balanced diet. C) Informing the client about follow-up sputum cultures D) Teaching the client ways to balance rest with activity

A, Educating the client on adherence to the treatment regimen.

An emergency department nurse assesses a client admitted after a lightning strike. Which assessment should the nurse complete first? A) Electrocardiogram (ECG). B) Wound inspection. C) Creatinine kinase. D) Computed tomography of head.

A, Electrocardiogram.

A nurse assesses a client who is prescribed fluticasone (Flovent) and notes oral lesions. Which action should the nurse take? A) Encourage oral rinsing after fluticasone administration. B) Obtain an oral specimen for culture and sensitivity. C) Start the client on a broad-spectrum antibiotic. D) Document the finding as a known side effect.

A, Encourage oral rinsing after fluticasone administration.

A client hospitalized for chemotherapy has a hemoglobin of 6.1 mg/dL. What medication should the nurse prepare to administer? A) Epoetin alfa (Epogen). B) Filgrastim (Neupogen). C) Mesna (Mesnex). D) Oprelvekin (Neumega).

A, Epoetin alfa.

The nurse working with oncology clients understands that interacting factors affect cancer development. Which factors does this include? (Select all that apply.) A) Exposure to carcinogens. B) Genetic predisposition. C) Immune function. D) Normal doubling time. E) State of euploidy.

A, Exposure to carcinogens. B, Genetic predisposition. C, Immune function.

A client has Crohns disease. What type of anemia is this client most at risk for developing? A) Folic acid deficiency B) Fanconis anemia C) Hemolytic anemia D) Vitamin B12 anemia

A, Folic acid deficiency.

The nurse is caring for four hypertensive clients. Which drug laboratory value combination should the nurse report immediately to the health care provider? A) Furosemide (Lasix)/potassium: 2.1 mEq/L B) Hydrochlorothiazide (Hydrodiuril)/potassium: 4.2 mEq/L C) Spironolactone (Aldactone)/potassium: 5.1 mEq/L D) Torsemide (Demadex)/sodium: 142 mEq/L

A, Furosemide (Lasix)/potassium: 2.1 mEq/L.

A client hospitalized with sickle cell crisis frequently asks for opioid pain medications, often shortly after receiving a dose. The nurses on the unit believe the client is drug seeking. When the client requests pain medication, what action by the nurse is best? A) Give the client pain medication if it is time for another dose. B) Instruct the client not to request pain medication too early. C) Request the provider leave a prescription for a placebo. D) Tell the client it is too early to have more pain medication.

A, Give the client pain medication if it is time for another dose.

A student nurse is helping a registered nurse with a blood transfusion. Which actions by the student are most appropriate? (Select all that apply.) A) Hanging the blood product using normal saline and a filtered tubing set B) Taking a full set of vital signs prior to starting the blood transfusion C) Telling the client someone will remain at the bedside for the first 5 minutes D) Using gloves to start the clients IV if needed and to handle the blood product E) Verifying the clients identity, and checking blood compatibility and expiration time

A, Hanging the blood product using normal saline and a filtered tubing set. B, Taking a full set of vital signs prior to starting the blood transfusion. D, Using gloves to start the clients IV if needed and to handle the blood product.

A nurse plans care for a client with epilepsy who is admitted to the hospital. Which interventions should the nurse include in this clients plan of care? (Select all that apply.) A) Have suction equipment at the bedside. B) Place a padded tongue blade at the bedside. C) Permit only clear oral fluids. D) Keep bed rails up at all times. E) Maintain the client on strict bedrest. F) Ensure that the client has IV access.

A, Have suction equipment at the bedside. D, Keep bed rails up at all times. F, Ensure that the client has IV access.

A nurse assesses a client who had a myocardial infarction and is hypotensive. Which additional assessment finding should the nurse expect? A) Heart rate of 120 beats/min B) Cool, clammy skin C) Oxygen saturation of 90% D) Respiratory rate of 8 breaths/min

A, Hear rate of 120 beats/min.

A nurse assesses a client who experienced a spinal cord injury at the T5 level 12 hours ago. Which manifestations should the nurse correlate with neurogenic shock? (Select all that apply.) A) Heart rate of 34 beats/min B) Blood pressure of 185/65 mm Hg C) Urine output less than 30 mL/hr D) Decreased level of consciousness E) Increased oxygen saturation

A, Heart rate of 34 beats/min. C, Urine output less than 30 mL/hr. D, Decreased level of consciousness.

A client is in shock and the nurse prepares to administer insulin for a blood glucose reading of 208 mg/dL. The spouse asks why the client needs insulin as the client is not a diabetic. What response by the nurse is best? A) High glucose is common in shock and needs to be treated. B) Some of the medications we are giving are to raise blood sugar. C) The IV solution has lots of glucose, which raises blood sugar. D) The stress of this illness has made your spouse a diabetic.

A, High glucose is common in shock and needs to be treated.

A nurse teaches a community health class about water safety. Which statement by a participant indicates that additional teaching is needed? A) I can go swimming all by myself because I am a certified lifeguard. B) I cannot leave my toddler alone in the bathtub for even a minute. C) I will appoint one adult to supervise the pool at all times during a party. D) I will make sure that there is a phone near my pool in case of an emergency.

A, I can go swimming all by myself because I am a certified lifeguard.

A client has peripheral arterial disease (PAD). What statement by the client indicates misunderstanding about self-management activities? A) I can use a heating pad on my legs if its set on low. B) I should not cross my legs when sitting or lying down. C) I will go out and buy some warm, heavy socks to wear. D) Its going to be really hard but I will stop smoking.

A, I can use a heating pad on my legs if its set on low.

An emergency room nurse obtains the health history of a client. Which statement by the client should alert the nurse to the occurrence of heart failure? A) I get short of breath when I climb stairs. B) I see halos floating around my head. C) I have trouble remembering things. D) I have lost weight over the past month.

A, I get short of breath when I climb stairs.

After teaching a client with congestive heart failure (CHF), the nurse assesses the clients understanding. Which client statements indicate a correct understanding of the teaching related to nutritional intake? (Select all that apply.) A) Ill read the nutritional labels on food items for salt content. B) I will drink at least 3 liters of water each day. C) Using salt in moderation will reduce the workload of my heart. D) I will eat oatmeal for breakfast instead of ham and eggs. E) Substituting fresh vegetables for canned ones will lower my salt intake.

A, Ill read the nutritional labels on food items for salt content. D, I will eat oatmeal for breakfast instead of ham and eggs. E, Substituting fresh vegetables for canned ones will lower my salt intake.

A nurse cares for a client who is experiencing epistaxis. Which action should the nurse take first? A) Initiate Standard Precautions. B) Apply direct pressure. C) Sit the client upright. D) Loosely pack the nares with gauze.

A, Initiate standard precautions.

A nurse assesses a client with tachycardia. Which clinical manifestation requires immediate intervention by the nurse? A) Mid-sternal chest pain B) Increased urine output C) Mild orthostatic hypotension D) P wave touching the T wave

A, Mid-sternal chest pain.

A nurse is teaching a larger female client about alcohol intake and how it affects hypertension. The client asks if drinking two beers a night is an acceptable intake. What answer by the nurse is best? A) No, women should only have one beer a day as a general rule. B) No, you should not drink any alcohol with hypertension. C) Yes, since you are larger, you can have more alcohol. D) Yes, two beers per day is an acceptable amount of alcohol.

A, No, women should only have one beer a day as a general rule.

A nurse assesses a client who is 6 hours post-surgery for a nasal fracture and has nasal packing in place. Which actions should the nurse take? (Select all that apply.) A) Observe for clear drainage. B) Assess for signs of bleeding. C) Watch the client for frequent swallowing. D) Ask the client to open his or her mouth. E) Administer a nasal steroid to decrease edema. F) Change the nasal packing.

A, Observe for clear drainage. B, Assess for signs of bleeding. C, Watch the client for frequent swallowing. D, Ask the client to open his or her mouth.

A nurse assesses a client recovering from a cerebral angiography via the clients right femoral artery. Which assessment should the nurse complete? A) Palpate bilateral lower extremity pulses. B) Obtain orthostatic blood pressure readings. C) Perform a funduscopic examination. D) Assess the gag reflex prior to eating.

A, Palpate bilateral lower extremity pulses.

Emergency medical services (EMS) brings a large number of clients to the emergency department following a mass casualty incident. The nurse identifies the clients with which injuries with yellow tags? (Select all that apply.) A) Partial-thickness burns covering both legs. B) Open fractures of both legs with absent pedal pulses. C) Neck injury and numbness of both legs. D) Small pieces of shrapnel embedded in both eyes. E) Head injury and difficult to arouse. F) Bruising and pain in the right lower abdomen.

A, Partial thickness burns covering both legs. C, Neck injury and numbness of both legs. D, Small pieces of shrapnel embedded in both eyes. F, Bruising and pain in the right lower abdomen.

A nurse plans care for a client with burn injuries. Which interventions should the nurse include in this clients plan of care to ensure adequatenutrition? (Select all that apply.) A) Provide at least 5000 kcal/day. B) Start an oral diet on the first day. C) Administer a diet high in protein. D) Collaborate with a registered dietitian. E) Offer frequent high-calorie snacks.

A, Provide at least 5000 kcal/day. C, Administer a diet high in protein. D, Collaborate with a registered dietician. E, Offer frequent high-calorie snacks.

A nurse is assessing a client with left-sided heart failure. For which clinical manifestations should the nurse assess? (Select all that apply.) A) Pulmonary crackles B) Confusion, restlessness C) Pulmonary hypertension D) Dependent edema E) Cough that worsens at night

A, Pulmonary crackles. B, Confusion and restlessness. E, Cough that worsens at night.

A nurse cares for a client with infective endocarditis. Which infection control precautions should the nurse use? A) Standard Precautions B) Bleeding precautions C) Reverse isolation D) Contact isolation

A, Standard precautions.

A nurse assesses a client who has facial trauma. Which assessment findings require immediate intervention? (Select all that apply.) A) Stridor B) Nasal stuffiness C) Edema of the cheek D) Ecchymosis behind the ear E) Eye pain F) Swollen chin

A, Stridor. D, Ecchymosis behind the ear.

A nurse is teaching a wilderness survival class. Which statements should the nurse include about the prevention of hypothermia and frostbite? (Select all that apply.) A) Wear synthetic clothing instead of cotton to keep your skin dry. B) Drink plenty of fluids. Brandy can be used to keep your body warm. C) Remove your hat when exercising to prevent the loss of heat. D) Wear sunglasses to protect skin and eyes from harmful rays. E) Know your physical limits. Come in out of the cold when limits are reached.

A, Wear synthetic clothing instead of cotton to keep your skin dry. C, Remove your hat when exercising to prevent the loss of heat. E, Know your physical limits.

A nurse cares for a client with right-sided heart failure. The client asks, Why do I need to weigh myself every day? How should the nurse respond? A) Weight is the best indication that you are gaining or losing fluid. B) Daily weights will help us make sure that your'e eating properly. C) The hospital requires that all inpatients be weighed daily. D) You need to lose weight to decrease the incidence of heart failure.

A, Weight is the best indication that you are gaining or losing fluid.

A nurse assesses a client after administering isosorbide mononitrate (Imdur). The client reports a headache. Which action should the nurse take?

Administer PRN acetaminophen.

A client has pernicious anemia. The nurse knows which of the following should be completed to increase their B12 levels?

Administer an IM or SubQ injection of B12.

A nurse receives new prescriptions for a client with severe burn injuries who is receiving fluid resuscitation per the Parkland formula. The clients urine output continues to range from 0.2 to 0.25 mL/kg/hr. Which prescription should the nurse question?

Administer furosemide (Lasix) 40 mg IV push.

A client requires administration of WBCs due to neutropenic status. What is the most important part of how a nurse administers this?

Administer slowly to monitor for reaction.

A nurse cares for a client with burn injuries. Which intervention should the nurse implement to appropriately reduce the clients pain?

Administer the prescribed intravenous morphine sulfate.

What race/ethnicity is at greatest risk of developing SCD?

African American.

A nurse is interested in providing community education and screening on hypertension. In order to reach a priority population, to what target audience should the nurse provide this service?

African-American churches.

A client with Vitamin B12 deficiency has been educated on foods to increase dietary intake. The nurse knows further education is required after the client states they will increase the intake of which food?

Alcohol. This decreased B12 levels.

A client is receiving norepinephrine (Levophed) for shock. What assessment finding best indicates a therapeutic effect from this drug?

Alert and oriented, answering questions.

A client is in the preoperative holding area prior to an emergency coronary artery bypass graft (CABG). The client is yelling at family members and tells the doctor to just get this over with when asked to sign the consent form. What action by the nurse is best?

All the client that anxiety is common and that you can help.

A client is in the hospital after suffering a myocardial infarction and has bathroom privileges. The nurse assists the client to the bathroom and notes the clients O2 saturation to be 95%, pulse 88 beats/min, and respiratory rate 16 breaths/min after returning to bed. What action by the nurse is best?

Allow continued bathroom privileges.

A nurse is caring for a client who is intubated and has an intra-aortic balloon pump. The client is restless and agitated. What action should the nurse perform first for comfort?

Allow the family members to remain at the bedside.

What are examples of K Channel Blockers?

Amiodarone and Droneadarone.

A nurse is reinforcing teaching about auras with a client who has a new diagnosis of simple partial seizures. Which of the following statements by the client indicates an understanding of the teaching?

An aura is a sensory warning that a seizure is imminent.

Where is S1 best heard?

Apex or PMI.

A client has been brought to the emergency department after being shot multiple times. What action should the nurse perform first?

Apply PPE.

An emergency room nurse assesses a client who was rescued from a home fire. The client suddenly develops a loud, brassy cough. Which action should the nurse take first?

Apply oxygen and continuous pulse oximetry.

A nurse assesses a client who has a history of heart failure.

Are you able to walk upstairs without fatigue?

A client is 4 hours postoperative after a femoropopliteal bypass. The client reports throbbing leg pain on the affected side, rated as 7/10. What action by the nurse takes priority?

Assess distal pulses and skin color.

An older adult is on cardiac monitoring after a myocardial infarction. The client shows frequent dysrhythmias. What action by the nurse is most appropriate?

Assess for any hemodynamic effects of the rhythm.

Upon auscultation of an elderly patient, the nurse heard an S3 heart sound. What is the most appropriate intervention by the nurse?

Assess for signs of fluid overload.

While assessing a client on a cardiac unit, a nurse identifies the presence of an S3 gallop. Which action should the nurse take next?

Assess for symptoms of left-sided heart failure.

A client is on a dopamine infusion via a peripheral line. What action by the nurse takes priority for safety?

Assess the IV site hourly.

A client has intra-arterial blood pressure monitoring after a myocardial infarction. The nurse notes the clients heart rate has increased from 88 to 110 beats/min, and the blood pressure dropped from 120/82 to 100/60 mm Hg. What action by the nurse is most appropriate?

Assess the client for bleeding.

A client with a history of heart failure and hypertension is in the clinic for a follow-up visit. The client is on lisinopril (Prinivil) and warfarin (Coumadin). The client reports new- onset cough. What action by the nurse is most appropriate?

Assess the client's lung sounds and oxygenation.

A client has hypertension and high risk factors for cardiovascular disease. The client is overwhelmed with the recommended lifestyle changes. What action by the nurse is best?

Assess the client's support system.

A nurse admits a client who is experiencing an exacerbation of heart failure. Which action should the nurse take first?

Assess the clients respiratory status.

A nurse is caring for a client after surgery. The clients respiratory rate has increased from 12 to 18 breaths/min and the pulse rate increased from 86 to 98 beats/min since they were last assessed 4 hours ago. What action by the nurse is best?

Assess the clients tissue perfusion further.

Upon auscultation of the aortic area, the nurse hears a murmur. What is the most appropriate intervention by the nurse?

Assess the patient's CO.

A client has been diagnosed with a deep vein thrombosis and is to be discharged on warfarin (Coumadin). The client is adamant about refusing the drug because its dangerous. What action by the nurse is best?

Assess the reason behind the client's fear.

What are Ca Channel Blockers used for?

Atrial dysrhythmias.

A nurse assesses a client who has mitral valve regurgitation. For which cardiac dysrhythmia should the nurse assess?

Atrial fibrillation.

A nurse cares for a client with a burn injury who presents with drooling and difficulty swallowing. Which action should the nurse take first?

Auscultate breath sounds over the trachea and bronchi.

Hemolytic anemia occurs by which of the following?

Autoimmune reaction.

A client has received education regarding hemophilia. Which of the following is most important to ensure the client understands?

Avoid contact sports or activities.

A nurse is reinforcing teaching with a client who has a new diagnosis of Meniere's disease. Which of the following instructions should the nurse include in the teaching?

Avoid sudden movements.

A nurse is teaching a client with heart failure who has been prescribed enalapril (Vasotec). Which statement should the nurse include in this clients teaching?

Avoid using salt substitutes.

A registered nurse (RN) cares for clients on a surgical unit. Which clients should the RN delegate to a licensed practical nurse (LPN)? (Select all that apply.) A) A 32-year-old who had a radical neck dissection 6 hours ago. B) A 43-year-old diagnosed with cancer after a lung biopsy 2 days ago. C) A 55-year-old who needs discharge teaching after a laryngectomy. D) A 67-year-old who is awaiting preoperative teaching for laryngeal cancer. E) An 88-year-old with esophageal cancer who is awaiting gastric tube placement

B, A 43-year-old diagnoses with cancer after a lung biopsy 2 days ago. E, An 88-year-old with esophageal cancer who is awaiting gastric tube placement.

A nurse is triaging clients in the emergency department (ED). Which client should the nurse prioritize to receive care first? A) A 22-year-old with a painful and swollen right wrist. B) A 45-year-old reporting chest pain and diaphoresis C) A 60-year-old reporting difficulty swallowing and nausea. D) An 81-year-old with a respiratory rate of 28 breaths/min and a temperature of 101 F.

B, A 45-year-old reporting chest pain and diaphoresis.

A pulmonary nurse cares for clients who have chronic obstructive pulmonary disease (COPD). Which client should the nurse assess first? A) A 46-year-old with a 30pack-year history of smoking. B) A 52-year-old in a tripod position using accessory muscles to breathe. C) A 68-year-old who has dependent edema and clubbed fingers. D) A 74-year-old with a chronic cough and thick, tenacious secretions

B, A 52-year-old in a tripod position using accessory muscles to breathe.

A client has been taking isoniazid (INH) for tuberculosis for 3 weeks. What laboratory results need to be reported to the health care provider immediately? A) Albumin: 5.1 g/dL B) Alanine aminotransferase (ALT): 180 U/L C) Red blood cell (RBC) count: 5.2/mm3 D) White blood cell (WBC) count: 12,500/mm3

B, ALT 180 U/L.

A nurse is assessing a client who is recovering from a lung biopsy. Which assessment finding requires immediate action? A) Increased temperature. B) Absent breath sounds. C) Productive cough. D) Incisional discomfort

B, Absent breath sounds.

A nurse receives new prescriptions for a client with severe burn injuries who is receiving fluid resuscitation per the Parkland formula. The clients urine output continues to range from 0.2 to 0.25 mL/kg/hr. Which prescription should the nurse question? A) Increase intravenous fluids by 100 mL/hr. B) Administer furosemide (Lasix) 40 mg IV push. C) Continue to monitor urine output hourly. D) Draw blood for serum electrolytes STAT.

B, Administer furosemide (Lasix) 40 mg IVP.

A client presents to the emergency department after prolonged exposure to the cold. The client is difficult to arouse and speech is incoherent. Which action should the nurse take first? A) Reposition the client into a prone position. B) Administer warmed intravenous fluids to the client. C) Wrap the clients extremities in warm blankets. D) Initiate extracorporeal rewarming via hemodialysis.

B, Administer warmed IV fluids to the client.

3. A client is in the hospital after suffering a myocardial infarction and has bathroom privileges. The nurse assists the client to the bathroom and notes the clients O2 saturation to be 95%, pulse 88 beats/min, and respiratory rate 16 breaths/min after returning to bed. What action by the nurse is best? A) Administer oxygen at 2 L/min. B) Allow continued bathroom privileges. C) Obtain a bedside commode. D) Suggest the client use a bedpan.

B, Allow continued bathroom privileges.

A client has a deep vein thrombosis (DVT). What comfort measure does the nurse delegate to the unlicensed assistive personnel (UAP)? A) Ambulate the client. B) Apply a warm moist pack. C) Massage the clients leg. D) Provide an ice pack.

B, Apply a warm moist pack.

The emergency department team is performing cardiopulmonary resuscitation on a client when the clients spouse arrives at the emergency department. Which action should the nurse take first? A) Request that the clients spouse sit in the waiting room. B) Ask the spouse if he wishes to be present during the resuscitation. C) Suggest that the spouse begin to pray for the client. D) Refer the clients spouse to the hospitals crisis team.

B, Ask the spouse if he wishes to be present during the resuscitation.

A client is 4 hours postoperative after a femoropopliteal bypass. The client reports throbbing leg pain on the affected side, rated as 7/10. What action by the nurse takes priority? A) Administer pain medication as ordered. B) Assess distal pulses and skin color. C) Document the findings in the clients chart. D) Notify the surgeon immediately.

B, Assess distal pulses and skin color.

A client has intra-arterial blood pressure monitoring after a myocardial infarction. The nurse notes the clients heart rate has increased from 88 to 110 beats/min, and the blood pressure dropped from 120/82 to 100/60 mm Hg. What action by the nurse is most appropriate? A) Allow the client to rest quietly. B) Assess the client for bleeding. C) Document the findings in the chart. D) Medicate the client for pain.

B, Assess the client for bleeding.

A nurse is caring for a client after surgery. The clients respiratory rate has increased from 12 to 18 breaths/min and the pulse rate increased from 86 to 98 beats/min since they were last assessed 4 hours ago. What action by the nurse is best? A) Ask if the client needs pain medication. B) Assess the clients tissue perfusion further. C) Document the findings in the clients chart. D) Increase the rate of the clients IV infusion.

B, Assess the clients tissue perfusion further.

A client has hypertension and high risk factors for cardiovascular disease. The client is overwhelmed with the recommended lifestyle changes. What action by the nurse is best? A) Assess the clients support system. B) Assist in finding one change the client can control. C) Determine what stressors the client faces in daily life. D) Inquire about delegating some of the clients obligations.

B, Assist in finding one change the client can control.

A client has received a dose of ondansetron (Zofran) for nausea. What action by the nurse is most important? A) Assess the client for a headache. B) Assist the client in getting out of bed. C) Instruct the client to reduce salt intake. D) Weigh the client daily before the client eats.

B, Assist the client in getting out of bed.

A client is 1 day postoperative after a coronary artery bypass graft. What nonpharmacologic comfort measures does the nurse include when caring for this client? (Select all that apply.) A) Administer pain medication before ambulating. B) Assist the client into a position of comfort in bed. C) Encourage high-protein diet selections. D) Provide complementary therapies such as music. E) Remind the client to splint the incision when coughing.

B, Assist the client into a position of comfort in bed. D, Provide complementary therapies such as music. E, Remind the client to splint the incision when coughing.

A nurse teaches a client recovering from a heart transplant who is prescribed cyclosporine (Sandimmune). Which statement should the nurse include in this clients discharge teaching? A) Use a soft-bristled toothbrush and avoid flossing. B) Avoid large crowds and people who are sick. C) Change positions slowly to avoid hypotension. D) Check your heart rate before taking the medication.

B, Avoid large crowds and people who are sick.

A nurse cares for a client who has a heart rate averaging 56 beats/min with no adverse symptoms. Which activity modification should the nurse suggest to avoid further slowing of the heart rate? A) Make certain that your bath water is warm. B) Avoid straining while having a bowel movement. C) Limit your intake of caffeinated drinks to one a day. D) Avoid strenuous exercise such as running.

B, Avoid straining while having a bowel movement.

A nurse cares for a client recovering from prosthetic valve replacement surgery. The client asks, Why will I need to take anticoagulants for the rest of my life? How should the nurse respond? A) The prosthetic valve places you at greater risk for a heart attack. B) Blood clots form more easily in artificial replacement valves. C) The vein taken from your leg reduces circulation in the leg. D) The surgery left a lot of small clots in your heart and lungs.

B, Blood clots form more easily in artificial replacement valves.

The nurse is caring for a client with an acute burn injury. Which action should the nurse take to prevent infection by autocontamination? A) Use a disposable blood pressure cuff to avoid sharing with other clients. B) Change gloves between wound care on different parts of the clients body. C) Use the closed method of burn wound management for all wound care. D) Advocate for proper and consistent handwashing by all members of the staff.

B, Change gloves between wound care on different parts of the clients body.

A student nurse is preparing to administer enoxaparin (Lovenox) to a client. What action by the student requires immediate intervention by the supervising nurse? A) Assessing the clients platelet count B) Choosing an 18-gauge, 2-inch needle C) Not aspirating prior to injection D) Swabbing the injection site with alcohol

B, Choosing an 18-gauge, 2-inch needle.

A nurse is caring for four clients. Which one should the nurse see first? A) Client who needs a beta blocker, and has a blood pressure of 92/58 mm Hg B) Client who had a first dose of captopril (Capoten) and needs to use the bathroom C) Hypertensive client with a blood pressure of 188/92 mm Hg D) Client who needs pain medication prior to a dressing change of a surgical wound

B, Client who had a first dose of captopril (Capoten) and needs to use the bathroom.

A nurse is caring for four clients. Which client should the nurse assess first? A) Client with an acute myocardial infarction, pulse 102 beats/min B) Client who is 1 hour post angioplasty, has tongue swelling and anxiety C) Client who is post coronary artery bypass, chest tube drained 100 mL/hr D) Client who is post coronary artery bypass, potassium 4.2 mEq/L

B, Client who is 1 hour post angioplasty, has tongue swelling and anxiety.

A nurse in a hematology clinic is working with four clients who have polycythemia vera. Which client should the nurse see first? A) Client with a blood pressure of 180/98 mm Hg B) Client who reports shortness of breath C) Client who reports calf tenderness and swelling D) Client with a swollen and painful left great toe

B, Client who reports shortness of breath.

A nurse is caring for five clients. For which clients would the nurse assess a high risk for developing a pulmonary embolism (PE)? (Select all that apply.) A) Client who had a reaction to contrast dye yesterday B) Client with a new spinal cord injury on a rotating bed C) Middle-aged man with an exacerbation of asthma D) Older client who is 1-day post hip replacement surgery E) Young obese client with a fractured femur

B, Client with a new spinal cord injury on a rotating bed. D, Older client who is 1-day post hip replacement surgery. E, Young obese client with a fractured femur.

While assessing a client who is 12 hours postoperative after a thoracotomy for lung cancer, a nurse notices that the lower chest tube is dislodged. Which action should the nurse take first? A) Assess for drainage from the site. B) Cover the insertion site with sterile gauze. C) Contact the provider and obtain a suture kit. D) Reinsert the tube using sterile technique.

B, Cover the insertion site with the sterile gauze.

A nurse assesses a client who has encephalitis. Which manifestations should the nurse recognize as signs of increased intracranial pressure (ICP), a complication of encephalitis? (Select all that apply.) A) Photophobia B) Dilated pupils C) Headache D) Widened pulse pressure E) Bradycardia

B, Dilated pupils. C, Headache. D, Widened pulse pressure.

A client tells the oncology nurse about an upcoming vacation to the beach to celebrate completing radiation treatments for cancer. What response by the nurse is most appropriate? A) Avoid getting salt water on the radiation site. B) Do not expose the radiation area to direct sunlight. C) Have a wonderful time and enjoy your vacation! D) Remember you should not drink alcohol for a year.

B, Do not expose the radiation area to direct sunlight.

A nurse obtains a focused health history for a client who is scheduled for magnetic resonance angiography. Which priority question should the nurse ask before the test? A) Have you had a recent blood transfusion? B) Do you have allergies to iodine or shellfish? C) Are you taking any cardiac medications? D) Do you currently use oral contraceptives?

B, Do you have allergies to iodine or shellfish?

A nurse assesses a client with chronic obstructive pulmonary disease. Which questions should the nurse ask to determine the clients activity tolerance? (Select all that apply.) A) What color is your sputum? B) Do you have any difficulty sleeping? C) How long does it take to perform your morning routine? D) Do you walk upstairs every day? E) Have you lost any weight lately?

B, Do you have any difficulty sleeping? C, How long does it take to perform your morning routine? E, Have you lost any weight recently?

client receiving a blood transfusion develops anxiety and low back pain. After stopping the transfusion, what action by the nurse is most important? A) Documenting the events in the clients medical record B) Double-checking the client and blood product identification C) Placing the client on strict bedrest until the pain subsides D) Reviewing the clients medical record for known allergies

B, Double-checking the client and blood product identification.

A nurse cares for a client with burn injuries from a house fire. The client is not consistently oriented and reports a headache. Which action should the nurse take? A) Increase the clients oxygen and obtain blood gases. B) Draw blood for a carboxyhemoglobin level. C) Increase the clients intravenous fluid rate. D) Perform a thorough Mini-Mental State Examination.

B, Draw blood for a carboxyhemoglobin level.

A client arrives in the emergency department after being in a car crash with fatalities. The client has a nearly amputated leg that is bleeding profusely. What action by the nurse takes priority? A) Apply direct pressure to the bleeding. B) Ensure the client has a patent airway. C) Obtain consent for emergency surgery. D) Start two large-bore IV catheters.

B, Ensure the client has a patent airway.

A client is having a bone marrow biopsy today. What action by the nurse takes priority? A) Administer pain medication first. B) Ensure valid consent is on the chart. C) Have the client shower in the morning. D) Premedicate the client with sedatives.

B, Ensure valid consent is on the chart.

A nurse is preparing to administer a blood transfusion. What action is most important? A) Correctly identifying client using two identifiers B) Ensuring informed consent is obtained if required C) Hanging the blood product with Ringers lactate D) Staying with the client for the entire transfusion

B, Ensuring informed consent is obtained if required.

A nurse is preparing to admit a client on mechanical ventilation from the emergency department. What action by the nurse takes priority? A) Assessing that the ventilator settings are correct. B) Ensuring there is a bag-valve-mask in the room. C) Obtaining personal protective equipment. D) Planning to suction the client upon arrival to the room

B, Ensuring there is a bag-valve-mask in the room.

A nurse assesses a client with pericarditis. Which assessment finding should the nurse expect to find? A) Heart rate that speeds up and slows down B) Friction rub at the left lower sternal border C) Presence of a regular gallop rhythm D) Coarse crackles in bilateral lung bases

B, Friction rub at the left lower sternal border.

A nurse teaches a client with heart failure about energy conservation. Which statement should the nurse include in this clients teaching? A) Walk until you become short of breath, and then walk back home. B) Gather everything you need for a chore before you begin. C) Pull rather than push or carry items heavier than 5 pounds. D) Take a walk after dinner every day to build up your strength.

B, Gather everything you need for a chore before you begin.

A client has presented to the emergency department with an acute myocardial infarction (MI). What action by the nurse is best to meet The Joint Commissions Core Measures outcomes? A) Obtain an electrocardiogram (ECG) now and in the morning. B) Give the client an aspirin. C) Notify the Rapid Response Team. D) Prepare to administer thrombolytics.

B, Give the client an aspirin.

A nurse assesses a client who has developed epistaxis. Which conditions in the clients history should the nurse identify as potential contributors to this problem? (Select all that apply.) A) Diabetes mellitus B) Hypertension C) Leukemia D) Cocaine use E) Migraine F) Elevated platelets

B, Hypertension. C, Leukemia. D, Cocaine use.

A client is being discharged home after a large myocardial infarction and subsequent coronary artery bypass grafting surgery. The clients sternal wound has not yet healed. What statement by the client most indicates a higher risk of developing sepsis after discharge? A) All my friends and neighbors are planning a party for me. B) I hope I can get my water turned back on when I get home. C) I am going to have my daughter scoop the cat litter box. D) My grandkids are so excited to have me coming home!

B, I hope I can get my water turned back on when I get home.

After teaching a client who has an implantable cardioverter-defibrillator (ICD), a nurse assesses the clients understanding. Which statement by the client indicates a correct understanding of the teaching? A) I should wear a snug-fitting shirt over the ICD. B) I will avoid sources of strong electromagnetic fields. C) I should participate in a strenuous exercise program. D) Now I can discontinue my antidysrhythmic medication.

B, I will avoid sources of strong electromagnetic fields.

After teaching a client who is being discharged home after mitral valve replacement surgery, the nurse assesses the clients understanding. Which client statement indicates a need for additional teaching? A) Ill be able to carry heavy loads after 6 months of rest. B) I will have my teeth cleaned by my dentist in 2 weeks. C) I must avoid eating foods high in vitamin K, like spinach. D) I must use an electric razor instead of a straight razor to shave.

B, I will have my teeth cleaned by my dentist in 2 weeks.

A client is on intravenous heparin to treat a pulmonary embolism. The clients most recent partial thromboplastin time (PTT) was 25 seconds. What order should the nurse anticipate? A) Decrease the heparin rate. B) Increase the heparin rate. C) No change to the heparin rate. D) Stop heparin; start warfarin (Coumadin).

B, Increase the heparin rate.

The student nurse caring for clients who have cancer understands that the general consequences of cancer include which client problems? (Select all that apply.) A) Clotting abnormalities from thrombocythemia. B) Increased risk of infection from white blood cell deficits. C) Nutritional deficits such as early satiety and cachexia D) Potential for reduced gas exchange. E) Various motor and sensory deficits.

B, Increased risk fo infection from white blood cell deficits. C, Nutritional deficits such as early satiety and cachexia. D, Potential for reduced gas exchange. E, Various motor and sensory deficits.

After administering newly prescribed captopril (Capoten) to a client with heart failure, the nurse implements interventions to decrease complications. Which priority intervention should the nurse implement for this client? A) Provide food to decrease nausea and aid in absorption. B) Instruct the client to ask for assistance when rising from bed. C) Collaborate with unlicensed assistive personnel to bathe the client. D) Monitor potassium levels and check for symptoms of hypokalemia.

B, Instruct the client to ask for assistance when rising from bed.

A client has a platelet count of 9800/mm3. What action by the nurse is most appropriate? A) Assess the client for calf pain, warmth, and redness. B) Instruct the client to call for help to get out of bed. C) Obtain cultures as per the facilitys standing policy. D) Place the client on protective isolation precautions.

B, Instruct the client to call for help to get out of bed.

A nurse is participating in primary prevention efforts directed against cancer. In which activities is this nurse most likely to engage? (Select all that apply.) A) Demonstrating breast self-examination methods to women. B) Instructing people on the use of chemoprevention. C) Providing vaccinations against certain cancers. D) Screening teenage girls for cervical cancer. E) Teaching teens the dangers of tanning booths.

B, Instructing people on the use of chem-prevention. C, Providing vaccinations against certain cancers. E, Teaching teens the dangers of tanning booths.

A provider prescribes diazepam (Valium) to a client who was bitten by a black widow spider. The client asks, What is this medication for? How should the nurse respond? A) This medication is an antivenom for this type of bite. B) It will relieve your muscle rigidity and spasms. C) It prevents respiratory difficulty from excessive secretions. D) This medication will prevent respiratory failure.

B, It will receive your muscle rigidity and spasms.

A nurse is caring for several clients at risk for shock. Which laboratory value requires the nurse to communicate with the health care provider? A) Creatinine: 0.9 mg/dL B) Lactate: 6 mmol/L C) Sodium: 150 mEq/L D) White blood cell count: 11,000/mm3

B, Lactate: 6 mmol/L.

.A client is being discharged soon on warfarin (Coumadin). What menu selection for dinner indicates the client needs more education regarding this medication? A) Hamburger and French fries B) Large chefs salad and muffin C) No selection; spouse brings pizza D) Tuna salad sandwich and chips

B, Large chefs salad and muffin.

A student is caring for a client who suffered massive blood loss after trauma. How does the student correlate the blood loss with the clients mean arterial pressure (MAP)? A) It causes vasoconstriction and increased MAP. B) Lower blood volume lowers MAP. C) There is no direct correlation to MAP. D) It raises cardiac output and MAP.

B, Lower blood volume lowers MAP.

A client has been diagnosed with hypertension but does not take the antihypertensive medications because of a lack of symptoms. What response by the nurse is best? A) Do you have trouble affording your medications? B) Most people with hypertension do not have symptoms. C) You are lucky; most people get severe morning headaches. D) You need to take your medicine or you will get kidney failure.

B, Most people with hypertension do not have symptoms.

A nurse obtains the health history of a client who is newly admitted to the medical unit. Which statement by the client should alert the nurse to the presence of edema? A) I wake up to go to the bathroom at night. B) My shoes fit tighter by the end of the day. C) I seem to be feeling more anxious lately. D) I drink at least eight glasses of water a day.

B, My shoes fit tighter by the end of the day.

A nurse assesses a client who is recovering from a lumbar puncture (LP). Which complication of this procedure should alert the nurse to urgently contact the health care provider? A) Weak pedal pulses B) Nausea and vomiting C) Increased thirst D) Hives on the chest

B, Nausea and vomiting.

A client is receiving an infusion of alteplase (Activase) for an intra-arterial clot. The client begins to mumble and is disoriented. What action by the nurse takes priority? A) Assess the clients neurologic status. B) Notify the Rapid Response Team. C) Prepare to administer vitamin K. D) Turn down the infusion rate.

B, Notify the Rapid Response Team.

The nurse is caring for a client with a chest tube after a coronary artery bypass graft. The drainage slows significantly. What action by the nurse is most important? A) Increase the setting on the suction. B) Notify the provider immediately. C) Re-position the chest tube. D) Take the tubing apart to assess for clots.

B, Notify the provider immediately.

A nurse answers a call light and finds a client anxious, short of breath, reporting chest pain, and having a blood pressure of 88/52 mm Hg on the cardiac monitor. What action by the nurse takes priority? A) Assess the clients lung sounds. B) Notify the Rapid Response Team. C) Provide reassurance to the client. D) Take a full set of vital signs.

B, Notify the rapid response team.

An older adult is brought to the emergency department by a family member, who reports a moderate change in mental status and mild cough. The client is afebrile. The health care provider orders a chest x-ray. The family member questions why this is needed since the manifestations seem so vague. What response by the nurse is best? A) Chest x-rays are always ordered when we suspect pneumonia. B) Older people often have vague symptoms, so an x-ray is essential. C) The x-ray can be done and read before laboratory work is reported. D) We are testing for any possible source of infection in the client.

B, Older people often have vague symptoms, so an x-ray is essential.

A nurse is caring for a client with a deep vein thrombosis (DVT). What nursing assessment indicates a priority outcome has been met? A) Ambulates with assistance B) Oxygen saturation of 98% C) Pain of 2/10 after medication D) Verbalizing risk factors

B, Oxygen saturation of 98%.

A nurse is caring for four clients on intravenous heparin therapy. Which laboratory value possibly indicates that a serious side effect has occurred? A) Hemoglobin: 14.2 g/dL B) Platelet count: 82,000/L C) Red blood cell count: 4.8/mm3 D) White blood cell count: 8.7/mm3

B, Platelet count 82,000/L.

On a hot humid day, an emergency department nurse is caring for a client who is confused and has these vital signs: temperature 104.1 F (40.1 C), pulse 132 beats/min, respirations 26 breaths/min, blood pressure 106/66 mm Hg. Which action should the nurse take? A) Encourage the client to drink cool water or sports drinks. B) Start an intravenous line and infuse 0.9% saline solution. C) Administer acetaminophen (Tylenol) 650 mg orally. D) Encourage rest and re-assess in 15 minutes.

B, Start an IV line and infuse 0..9% saline solution.

A nurse is teaching a client who experiences migraine headaches and is prescribed a beta blocker. Which statement should the nurse include in this clients teaching? A) Take this drug only when you have prodromal symptoms indicating the onset of a migraine headache. B) Take this drug as ordered, even when feeling well, to prevent vascular changes associated with migraine headaches. C) This drug will relieve the pain during the aura phase soon after a headache has started. D) This medication will have no effect on your heart rate or blood pressure because you are taking it for migraines.

B, Take this drug as ordered, even when feeling well, to prevent vascular changes associated with migraine headaches.

After teaching a client how to perform diaphragmatic breathing, the nurse assesses the clients understanding. Which action demonstrates that the client correctly understands the teaching? A) The client lays on his or her side with his or her knees bent. B) The client places his or her hands on his or her abdomen. C) The client lays in a prone position with his or her legs straight. D) The client places his or her hands above his or her head.

B, The client places his or hands on his or her abdomen.

A nurse assesses a client who has a chest tube. For which manifestations should the nurse immediately intervene? (Select all that apply.) A) Production of pink sputum B) Tracheal deviation C) Sudden onset of shortness of breath D) Pain at insertion site E) Drainage of 75 mL/hr

B, Tracheal deviation. C, Sudden onset of shortness of breath.

A nurse assesses a client who has a mediastinal chest tube. Which symptoms require the nurses immediate intervention? (Select all that apply.) A) Production of pink sputum B) Tracheal deviation C) Pain at insertion site D) Sudden onset of shortness of breath E) Drainage greater than 70 mL/hr F) Disconnection at Y site

B, Tracheal deviation. D, Sudden onset of shortness of breath. E, Drainage greater than 70 mL/hr. F, Disconnection at Y site.

A nurse witnesses a client begin to experience a tonic-clonic seizure and loss of consciousness. Which action should the nurse take? A) Start fluids via a large-bore catheter. B) Turn the clients head to the side. C) Administer IV push diazepam. D) Prepare to intubate the client.

B, Turn the clients head to the side.

A nurse assesses a client admitted with deep partial-thickness and full- thickness burns on the face, arms, and chest. Which assessment finding should alert the nurse to a potential complication? A) Partial pressure of arterial oxygen (PaO2) of 80 mm Hg. B) Urine output of 20 mL/hr. C) Productive cough with white pulmonary secretions. D) Core temperature of 100.6 F (38 C).

B, Urine output of 20 mL/hr.

A nurse evaluates prescriptions for a client with chronic atrial fibrillation. Which medication should the nurse expect to find on this clients medication administration record to prevent a common complication of this condition? A) Sotalol (Betapace) B) Warfarin (Coumadin) C) Atropine (Sal-Tropine) D) Lidocaine (Xylocaine)

B, Warfarin.

Where is the pulmonary valve located?

Between the 2nd ICS on the left sternal border.

Where is the aortic valve located?

Between the 2nd ICS on the right sternal border.

Where is the mitral valve (PMI) located?

Between the 5th ICS and mid-clavicular.

A nurse cares for a client recovering from prosthetic valve replacement surgery. The client asks, Why will I need to take anticoagulants for the rest of my life? How should the nurse respond?

Blood clots form more easily in artificial replacement valves.

Iron deficiency anemia is caused by which of the following?

Blood loss, poor GI absorption of iron, and inadequate diet. The diet lacks in iron fortified foods, dark green veggies, and red meats.

What is Atropine Sulfate used for?

Bradycardia.

A nurse uses the rule of nines to assess a client with burn injuries to the entire back region and left arm. How should the nurse document the percentage of the clients body that sustained burns? A) 9%. B) 18%. C) 27%. D) 36%.

C, 27%.

An emergency room nurse is triaging victims of a multi-casualty event. Which client should receive care first? A) A 30-year-old distraught mother holding her crying child. B) A 65-year-old conscious male with a head laceration. C) A 26-year-old male who has pale, cool, clammy skin. D) A 48-year-old with a simple fracture of the lower leg.

C, A 26-year old male who has pale, cool, clammy skin.

A nurse assesses clients on a medical-surgical unit. Which client should the nurse identify as having the greatest risk for cardiovascular disease? A) An 86-year-old man with a history of asthma B) A 32-year-old Asian-American man with colorectal cancer C) A 45-year-old American Indian woman with diabetes mellitus D) A 53-year-old postmenopausal woman who is on hormone therapy

C, A 45-year-old American Indian woman with diabetes mellitus.

A nurse assesses clients on the medical-surgical unit. Which client is at greatest risk for development of obstructive sleep apnea? A) A 26-year-old woman who is 8 months pregnant. B) A 42-year-old man with gastroesophageal reflux disease. C) A 55-year-old woman who is 50 pounds overweight. D) A 73-year-old man with type 2 diabetes mellitus

C, A 55-year-old woman who is 50 pounds overweight.

A nurse is triaging clients in the emergency department. Which client should the nurse classify as nonurgent? A) A 44-year-old with chest pain and diaphoresis. B) A 50-year-old with chest trauma and absent breath sounds. C) A 62-year-old with a simple fracture of the left arm. D) A 79-year-old with a temperature of 104 F.

C, A 62-year-old with a simple fracture of the left arm.

A nurse is triaging clients in the emergency department. Which client should be considered urgent? A) A 20-year-old female with a chest stab wound and tachycardia. B) A 45-year-old homeless man with a skin rash and sore throat. C) A 75-year-old female with a cough and a temperature of 102 F. D) A 50-year-old male with new-onset confusion and slurred speech.

C, A 75-year-old female with a cough and temperature of 102 F.

A nurse assesses a client with asthma and notes bilateral wheezing, decreased pulse oxygen saturation, and suprasternal retraction on inhalation. Which actions should the nurse take? (Select all that apply.) A) Administer prescribed salmeterol (Serevent) inhaler. B) Assess the client for a tracheal deviation. C) Administer oxygen to keep saturations greater than 94%. D) Perform peak expiratory flow readings. E) Administer prescribed albuterol (Proventil) inhaler.

C, Administer oxygen to keep saturations greater than 94%. E, Administer prescribed albuterol inhaler.

A provider prescribes Crotalidae Polyvalent Immune Fab (CroFab) for a client who is admitted after being bitten by a pit viper snake. Which assessment should the nurse complete prior to administering this medication? A) Assess temperature and for signs of fever. B) Check the clients creatinine kinase level. C) Ask about allergies to pineapple or papaya. D) Inspect the skin for signs of urticaria (hives).

C, Ask about allergies to pineapple or papaya.

A nurse cares for a client who had a bronchoscopy 2 hours ago. The client asks for a drink of water. Which action should the nurse take next? A) Call the physician and request a prescription for food and water. B) Provide the client with ice chips instead of a drink of water. C) Assess the clients gag reflex before giving any food or water. D) Let the client have a small sip to see whether he or she can swallow.

C, Assess the clients gag reflex before giving any food or water.

A nurse assesses an older adult client who has multiple chronic diseases. The clients heart rate is 48 beats/min. Which action should the nurse take first? A) Document the finding in the chart. B) Initiate external pacing. C) Assess the clients medications. D) Administer 1 mg of atropine.

C, Assess the clients medications.

A nurse cares for a client who is scheduled for a total laryngectomy. Which action should the nurse take prior to surgery? A) Assess airway patency, breathing, and circulation. B) Administer prescribed intravenous pain medication. C) Assist the client to choose a communication method. D) Ambulate the client in the hallway to assess gait.

C, Assist the client to choose a communication method.

A nurse assesses a client recently bitten by a coral snake. Which assessment should the nurse complete first? A) Unilateral peripheral swelling. B) Clotting times. C) Cardiopulmonary status. D) Electrocardiogram rhythm.

C, Cardiopulmonary status.

A nurse is caring for four clients. After reviewing todays laboratory results, which client should the nurse see first? A) Client with an international normalized ratio of 2.8 B) Client with a platelet count of 128,000/mm3 C) Client with a prothrombin time (PT) of 28 seconds D) Client with a red blood cell count of 5.1 million/L

C, Client with a prothrombin time (PT) of 28 seconds.

A nurse assesses an older adult client who is experiencing a myocardial infarction. Which clinical manifestation should the nurse expect? A) Excruciating pain on inspiration B) Left lateral chest wall pain C) Disorientation and confusion D) Numbness and tingling of the arm

C, Disorientation and confusion.

A nurse cares for a client with chronic obstructive pulmonary disease (COPD) who appears thin and disheveled. Which question should the nurse ask first? A) Do you have a strong support system? B) What do you understand about your disease? C) Do you experience shortness of breath with basic activities? D) What medications are you prescribed to take each day?

C, Do you experience shortness of breath with basic activities?

A client has been brought to the emergency department with a life-threatening chest injury. What action by the nurse takes priority? A) Apply oxygen at 100%. B) Assess the respiratory rate. C) Ensure a patent airway. D) Start two large-bore IV lines.

C, Ensure patent airway.

When working with women who are taking hormonal birth control, what health promotion measures should the nurse teach to prevent possible pulmonary embolism (PE)? (Select all that apply.) A) Avoid drinking alcohol. B) Eat more omega-3 fatty acids. C) Exercise on a regular basis. D) Maintain a healthy weight. E) Stop smoking cigarettes.

C, Exercise on a regular basis. D, Maintain a healthy weight. E, Stop smoking cigarettes.

An emergency room nurse assesses a client who has been raped. With which health care team member should the nurse collaborate when planning this clients care? A) Emergency medicine physician. B) Case manager. C) Forensic nurse examiner. D) Psychiatric crisis nurse.

C, Forensic nurse examiner.

A client is admitted with superior vena cava syndrome. What action by the nurse is most appropriate? A) Administer a dose of allopurinol (Aloprim). B) Assess the clients serum potassium level. C) Gently inquire about advance directives. D) Prepare the client for emergency surgery.

C, Gently inquire about advanced directives.

The nursing instructor explains the difference between normal cells and benign tumor cells. What information does the instructor provide about these cells? A) Benign tumors grow through invasion of other tissue. B) Benign tumors have lost their cellular regulation from contact inhibition. C) Growing in the wrong place or time is typical of benign tumors. D) The loss of characteristics of the parent cells is called anaplasia.

C, Growing in the wrong place or time is typical fo benign tumors.

A nurse assesses a client in an outpatient clinic. Which statement alerts the nurse to the possibility of left-sided heart failure? A) I have been drinking more water than usual. B) I am awakened by the need to urinate at night. C) I must stop halfway up the stairs to catch my breath. D) I have experienced blurred vision on several occasions.

C, I must stop halfway up the stairs to catch my breath.

The nurse is teaching a client with chronic obstructive pulmonary disease who has been prescribed continuous oxygen therapy at home. Which statement indicates the client correctly understands the teaching? A) I plan to wear my oxygen when I exercise and feel short of breath. B) I will use my portable oxygen when grilling burgers in the backyard. C) I plan to use cotton balls to cushion the oxygen tubing on my ears. D) I will only smoke while I am wearing my oxygen via nasal cannula.

C, I plan to use cotton balls to cushion the oxygen tubing on my ears.

A nurse assesses a client who has a burn injury. Which statement indicates the client has a positive perspective of his or her appearance? A) I will allow my spouse to change my dressings. B) I want to have surgical reconstruction. C) I will bathe and dress before breakfast. D) I have secured the pressure dressings as ordered.

C, I will bathe and dress before breakfast.

Which statements by the client indicate good understanding of foot care in peripheral vascular disease? (Select all that apply.) A) A good abrasive pumice stone will keep my feet soft. B) Ill always wear shoes if I can buy cheap flip-flops. C) I will keep my feet dry, especially between the toes. D) Lotion is important to keep my feet smooth and soft. E) Washing my feet in room-temperature water is best.

C, I will keep my feet dry, especially between the toes. D, Lotion is important to keep my feet smooth and soft. E, Washing my feet in room-temperature water is best.

A nurse has educated a client on isoniazid (INH). What statement by the client indicates teaching has been effective? A) I need to take extra vitamin C while on INH. B) I should take this medicine with milk or juice. C) I will take this medication on an empty stomach. D) My contact lenses will be permanently stained.

C, I will take this medication on an empty stomach.

7. After teaching a client who is prescribed salmeterol (Serevent), the nurse assesses the clients understanding. Which statement by the client indicates a need for additional teaching? A) I will be certain to shake the inhaler well before I use it. B) It may take a while before I notice a change in my asthma. C) I will use the drug when I have an asthma attack. D) I will be careful not to let the drug escape out of my nose and mouth.

C, I will use the drug when I have an asthma attack.

A nurse has taught a client about dietary changes that can reduce the chances of developing cancer. What statement by the client indicates the nurse needs to provide additional teaching? A) Foods high in vitamin A and vitamin C are important. B) Ill have to cut down on the amount of bacon I eat. C) I'm so glad I don't have to give up my juicy steaks. D) Vegetables, fruit, and high-fiber grains are important.

C, I'm so glad I don't have to give up my juicy steaks.

A client is admitted with suspected pneumonia from the emergency department. The client went to the primary care provider a few days ago and shows the nurse the results of what the client calls an allergy test, as shown below: What action by the nurse takes priority? A) Assess the client for possible items to which he or she is allergic. B) Call the primary care providers office to request records. C) Immediately place the client on Airborne Precautions. D) Prepare to begin administration of intravenous antibiotics.

C, Immediately place the client on airborne precautions.

A nurse is assisting the health care provider who is intubating a client. The provider has been attempting to intubate for 40 seconds. What action by the nurse takes priority? A) Ensure the client has adequate sedation. B) Find another provider to intubate. C) Interrupt the procedure to give oxygen. D) Monitor the clients oxygen saturation.

C, Interrupt the procedure to give oxygen.

A nurse prepares to administer intravenous cimetidine (Tagamet) to a client who has a new burn injury. The client asks, Why am I taking this medication? How should the nurse respond? A) Tagamet stimulates intestinal movement so you can eat more. B) It improves fluid retention, which helps prevent hypovolemic shock. C) It helps prevent stomach ulcers, which are common after burns. D) Tagamet protects the kidney from damage cause by dehydration.

C, It helps prevent stomach ulcers, which are common after burns.

The nurse is caring for a client with lung cancer who states, I dont want any pain medication because I am afraid Ill become addicted. How should the nurse respond? A) I will ask the provider to change your medication to a drug that is less potent. B) Would you like me to use music therapy to distract you from your pain? C) It is unlikely you will become addicted when taking medicine for pain. D) Would you like me to give you acetaminophen (Tylenol) instead?

C, It is unlikely you will become addicted when taking medicine for pain.

A client has a sickle cell crisis with extreme lower extremity pain. What comfort measure does the nurse delegate to the unlicensed assistive personnel (UAP)? A) Apply ice packs to the clients legs. B) Elevate the clients legs on pillows. C) Keep the lower extremities warm. D) Place elastic bandage wraps on the clients legs.

C, Keep the lower extremities warm.

An intubated clients oxygen saturation has dropped to 88%. What action by the nurse takes priority? A) Determine if the tube is kinked. B) Ensure all connections are patent. C) Listen to the clients lung sounds. D) Suction the endotracheal tube.

C, Listen to the clients lungs.

A nurse obtains the health history of a client who is recently diagnosed with lung cancer and identifies that the client has a 60pack-year smoking history. Which action is most important for the nurse to take when interviewing this client? A) Tell the client that he needs to quit smoking to stop further cancer development. B) Encourage the client to be completely honest about both tobacco and marijuana use. C) Maintain a nonjudgmental attitude to avoid causing the client to feel guilty. D) Avoid giving the client false hope regarding cancer treatment and prognosis.

C, Maintain a nonjudgemental attitude to acid causing the client to feel guilty.

A client in the cardiac stepdown unit reports severe, crushing chest pain accompanied by nausea and vomiting. What action by the nurse takes priority? A) Administer an aspirin. B) Call for an electrocardiogram (ECG). C) Maintain airway patency. D) Notify the provider.

C, Maintain airway patency.

A nurse caring for a client notes the following assessments: white blood cell count 3800/ mm3, blood glucose level 198 mg/dL, and temperature 96.2 F (35.6 C). What action by the nurse takes priority? A) Document the findings in the clients chart. B) Give the client warmed blankets for comfort. C) Notify the health care provider immediately. D) Prepare to administer insulin per sliding scale.

C, Notify the health care provider immediately.

A nurse assesses a client with a spinal cord injury at level T5. The clients blood pressure is 184/95 mm Hg, and the client presents with a flushed face and blurred vision. Which action should the nurse take first? A) Initiate oxygen via a nasal cannula. B) Place the client in a supine position. C) Palpate the bladder for distention. D) Administer a prescribed beta blocker.

C, Palpate the bladder for distention.

A client had an acute myocardial infarction. What assessment finding indicates to the nurse that a significant complication has occurred? A) Blood pressure that is 20 mm Hg below baseline B) Oxygen saturation of 94% on room air C) Poor peripheral pulses and cool skin D) Urine output of 1.2 mL/kg/hr for 4 hours

C, Poor peripheral pulses and cool skin.

A client in shock has been started on dopamine. What assessment finding requires the nurse to communicate with the provider immediately? A) Blood pressure of 98/68 mm Hg B) Pedal pulses 1+/4+ bilaterally C) Report of chest heaviness D) Urine output of 32 mL/hr

C, Report of chest heaviness .

A nurse reviews the laboratory results for a client who was burned 24 hours ago. Which laboratory result should the nurse report to the health care provider immediately? A) Arterial pH: 7.32 B) Hematocrit: 52% C) Serum potassium: 6.5 mEq/L D) Serum sodium: 131 mEq/L

C, Serum potassium 6.5 mEq/L.

A nurse cares for a client with chronic obstructive pulmonary disease (COPD). The client states that he no longer enjoys going out with his friends. How should the nurse respond? A) There are a variety of support groups for people who have COPD. B) I will ask your provider to prescribe you with an antianxiety agent. C) Share any thoughts and feelings that cause you to limit social activities. D) Friends can be a good support system for clients with chronic disorders.

C, Share and thoughts and feelings that cause you to limit social activities.

A nurse administers prescribed adenosine (Adenocard) to a client. Which response should the nurse assess for as the expected therapeutic response? A) Decreased intraocular pressure B) Increased heart rate C) Short period of asystole D) Hypertensive crisis

C, Short period of asystole.

A nurse cares for a female client who has a family history of cystic fibrosis. The client asks, Will my children have cystic fibrosis? How should the nurse respond? A) Since many of your family members are carriers, your children will also be carriers of the gene. B) Cystic fibrosis is an autosomal recessive disorder. If you are a carrier, your children will have the disorder. C) Since you have a family history of cystic fibrosis, I would encourage you and your partner to be tested. D) Cystic fibrosis is caused by a protein that controls the movement of chloride. Adjusting your diet will decrease the spread of this disorder.

C, Since you have a family history of CF, I would encourage you and your partner to be tested.

A nurse assesses a client who is recovering after a left-sided cardiac catheterization. Which assessment finding requires immediate intervention? A) Urinary output less than intake B) Bruising at the insertion site C) Slurred speech and confusion D) Discomfort in the left leg

C, Slurred speech and confusion.

A nurse teaches a client who is prescribed nicotine replacement therapy. Which statement should the nurse include in this clients teaching? A) Make a list of reasons why smoking is a bad habit. B) Rise slowly when getting out of bed in the morning. C) Smoking while taking this medication will increase your risk of a stroke. D) Stopping this medication suddenly increases your risk for a heart attack.

C, Smoking while taking this medication will increase your risk of a stroke.

The nurse teaches burn prevention to a community group. Which statement by a member of the group should cause the nurse the greatest concern? A) I get my chimney swept every other year. B) My hot water heater is set at 120 degrees. C) Sometimes I wake up at night and smoke. D) I use a space heater when it gets below zero.

C, Sometimes I wake up at night and smoke.

A nurse is caring for a young male client with lymphoma who is to begin treatment. What teaching topic is a priority? A) Genetic testing B) Infection prevention C) Sperm banking D) Treatment options

C, Sperm banking.

A client is receiving an infusion of tissue plasminogen activator (t-PA). The nurse assesses the client to be disoriented to person, place, and time. What action by the nurse is best? A) Assess the clients pupillary responses. B) Request a neurologic consultation. C) Stop the infusion and call the provider. D) Take and document a full set of vital signs.

C, Stop the infusion and call the provider.

A client has a pulmonary embolism and is started on oxygen. The student nurse asks why the clients oxygen saturation has not significantly improved. What response by the nurse is best? A) Breathing so rapidly interferes with oxygenation. B) Maybe the client has respiratory distress syndrome. C) The blood clot interferes with perfusion in the lungs. D) The client needs immediate intubation and mechanical ventilation.

C, The blood clot interferes with perfusion in the lungs.

While triaging clients in a crowded emergency department, a nurse assesses a client who presents with symptoms of tuberculosis. Which action should the nurse take first? A) Apply oxygen via nasal cannula. B) Administer intravenous 0.9% saline solution. C) Transfer the client to a negative-pressure room. D) Obtain a sputum culture and sensitivity.

C, Transfer the client to a negative-pressure room.

A nurse assesses a client who has a history of migraines. Which clinical manifestation should the nurse identify as an early sign of a migraine with aura? A) Vertigo B) Lethargy C) Visual disturbances D) Numbness of the tongue

C, Visual disturbances.

A nurse is providing care after auscultating clients breath sounds. Which assessment finding is correctly matched to the nurses primary intervention? A) Hollow sounds are heard over the trachea. The nurse increases the oxygen flow rate. B) Crackles are heard in bases. The nurse encourages the client to cough forcefully. C) Wheezes are heard in central areas. The nurse administers an inhaled bronchodilator. D) Vesicular sounds are heard over the periphery. The nurse has the client breathe deeply.

C, Wheezes are heard in central area. The nurse administers an inhaled bronchodilator.

A nurse cares for a client who has facial burns. The client asks, Will I ever look the same? How should the nurse respond? A) With reconstructive surgery, you can look the same. B) We can remove the scars with the use of a pressure dressing. C) You will not look exactly the same but cosmetic surgery will help. D) You shouldn't start worrying about your appearance right now.

C, You will not look exactly the same but cosmetic surgery will help.

A nurse cares for a client who tests positive for alpha1-antitrypsin (AAT) deficiency. The client asks, What does this mean? How should the nurse respond? A) Your children will be at high risk for the development of chronic obstructive pulmonary disease. B) I will contact a genetic counselor to discuss your condition. C) Your risk for chronic obstructive pulmonary disease is higher, especially if you smoke. D) This is a recessive gene and should have no impact on your health.

C, Your risk for COPD is higher, especially if you smoke.

A client has thrombocytopenia. What client statement indicates the client understands self-management of this condition? A) I brush and use dental floss every day. B) I chew hard candy for my dry mouth. C) I usually put ice on bumps or bruises. D) Nonslip socks are best when I walk.

C. I usually put ice on bumps or bruises.

A nurse assesses a client 2 hours after a cardiac angiography via the left femoral artery. The nurse notes that the left pedal pulse is weak. Which action should the nurse take? A) Elevate the leg and apply a sandbag to the entrance site. B) Increase the flow rate of intravenous fluids. C) Assess the color and temperature of the left leg. D) Document the finding as left pedal pulse of +1/4

C< Assess the color and temperature of the leg.

Secondary prevention of cancer?

Catch is early. Screenings.

What is the pathophysiology of cancer?

Cells lose their normal growth controlling mechanism and grow out of control. Malignant neoplastic disorder that can involve all body organs with s/s varying depending on type/stage of tumor cells. Can cause serious health problems such as impaired immune system, blood producing function, altered GI function, motor and sensory deficits, and decreased respiratory function.

The nurse is caring for a client with an acute burn injury. Which action should the nurse take to prevent infection by autocontamination?

Change gloves between wound care on different parts of the clients body.

Which of the following would represent correct maintenance of continuous TPN and lipids (pt is NPO)?

Change the TPN and lipid bag and tubing every 24 hours.

What is Adenosine used for?

Chemical cardioverter- Stable/certain unstable tachycardia with a pulse.

A nurse is collecting data from a client who is unconscious and has a rhythmical breathing pattern of rapid deep respirations, followed by rapid shallow respirations, alternating with periods of apnea. The nurse should document that the client is experiencing which of the following types of respirations?

Cheyne-Strokes.

A nurse is caring for four clients. Which one should the nurse see first?

Client who had first dose of Captopril and needs to use the bathroom.

A nurse is in charge of the coronary intensive care unit. Which client should the nurse see first?

Client who is 1 day post coronary artery bypass graft, blood pressure 180/100 mm Hg.

A nurse is caring for four clients. Which client should the nurse assess first?

Client who is 1 hour post angioplasty, has tongue swelling and anxiety.

The nurse gets the hand-off report on four clients. Which client should the nurse assess first?

Client with a blood pressure change of 128/74 to 110/88 mm Hg.

A client has ITP. Which of the following orders would the nurse question?

Clopidigrel. This is an anti-platelet that would increase risk of bleeding.

Patients with polycythemia vera are at increased risk of which of the following?

Clotting.

Pregnancy in women with SCD can be life threatening. What type of contraception would be contraindicated for this client?

Combination hormone therapy.

A nursing student is caring for a client who had a myocardial infarction. The student is confused because the client states nothing is wrong and yet listens attentively while the student provides education on lifestyle changes and healthy menu choices. What response by the faculty member is best?

Continue to educate the client on possible healthy changes.

A nurse is caring for client with hemochromatosis in a longterm care faciltiy that has an order for phlebotomy to remove 500mL of blood twice a week until ferritin levels reach normal levels. What is the most appropriate action by the nurse at this time?

Continue to monitor the client.

A client with sickle cell crisis may experience clumping of cells in arteries, especially those along joints. What S/S would the nurse note if this happened?

Cool or pale skin distal to site.

A nurse is assessing a client with peripheral artery disease (PAD). The client states walking five blocks is possible without pain. What question asked next by the nurse will give the best information?

Could you walk further than that a few months ago?

A nurse administers topical gentamicin sulfate (Garamycin) to a clients burn injury. Which laboratory value should the nurse monitor while the client is prescribed this therapy?

Creatinine.

A nurse assesses several clients who have a history of asthma. Which client should the nurse assess first? A) A 66-year-old client with a barrel chest and clubbed fingernails. B) A 48-year-old client with an oxygen saturation level of 92% at rest. C) A 35-year-old client who has a longer expiratory phase than inspiratory phased. D) A 27-year-old client with a heart rate of 120 beats/min

D, A 27-year-old client with a heart rate of 120 beats/min.

An emergency department nurse triages clients who present with chest discomfort. Which client should the nurse plan to assess first? A) A 42-year-old female who describes her pain as a dull ache with numbness in her fingers B) A 49-year-old male who reports moderate pain that is worse on inspiration C) A 53-year-old female who reports substernal pain that radiates to her abdomen D) A 58-year-old male who describes his pain as intense stabbing that spreads across his chest

D, A 58-year-old male who describes his pain as intense stabbing that spreads across his chest.

While at a public park, a nurse encounters a person immediately after a bee sting. The persons lips are swollen, and wheezes are audible. Which action should the nurse take first? A) Elevate the site and notify the persons next of kin. B) Remove the stinger with tweezers and encourage rest. C) Administer diphenhydramine (Benadryl) and apply ice. D) Administer an EpiPen from the first aid kit and call 911.

D, Administer an EpiPen from the first aid kit and call 911.

A nurse cares for a client who had a chest tube placed 6 hours ago and refuses to take deep breaths because of the pain. Which action should the nurse take? A) Ambulate the client in the hallway to promote deep breathing. B) Auscultate the clients anterior and posterior lung fields. C) Encourage the client to take shallow breaths to help with the pain. D) Administer pain medication and encourage the client to take deep breaths.

D, Administer pain medication and encourage the client to take deep breaths.

A nurse is assessing a client who has suffered a nasal fracture. Which assessment should the nurse perform first? A) Facial pain. B) Vital signs. C) Bone displacement. D) Airway patency.

D, Airway patency.

A nurse assesses a client who is scheduled for a cardiac catheterization. Which assessment should the nurse complete prior to this procedure? A) Clients level of anxiety B) Ability to turn self in bed C) Cardiac rhythm and heart rate D) Allergies to iodine-based agents

D, Allergies to iodine-based agent.s

A nurse teaches a client who is prescribed digoxin (Lanoxin) therapy. Which statement should the nurse include in this clients teaching? A) Avoid taking aspirin or aspirin-containing products. B) Increase your intake of foods that are high in potassium. C) Hold this medication if your pulse rate is below 80 beats/min. D) Do not take this medication within 1 hour of taking an antacid.

D, Do not take this medication within 1 hour of taking an antacid.

An emergency room nurse initiates care for a client with a cervical spinal cord injury who arrives via emergency medical services. Which action should the nurse take first? A) Assess level of consciousness. B) Obtain vital signs. C) Administer oxygen therapy. D) Evaluate respiratory status.

D, Evaluate respiratory status.

A nurse assesses bilateral wheezes in a client with burn injuries inside the mouth. Four hours later the wheezing is no longer heard. Which action should the nurse take? A) Document the findings and reassess in 1 hour. B) Loosen any constrictive dressings on the chest. C) Raise the head of the bed to a semi-Fowlers position. D) Gather appropriate equipment and prepare for an emergency airway.

D, Gather appropriate equipment and prepare for an emergency airway.

A nurse evaluates the following arterial blood gas and vital sign results for a client with chronic obstructive pulmonary disease (COPD):Arterial Blood Gas Results Vital SignspH = 7.32 PaCO2 = 62 mm Hg PaO2 = 46 mm Hg HCO3 = 28 mEq/L Heart rate = 110 beats/min Respiratory rate = 12 breaths/min Blood pressure = 145/65 mm HgOxygen saturation = 76% Which action should the nurse take first? A) Administer a short-acting beta2 agonist inhaler. B) Document the findings as normal for a client with COPD. C) Teach the client diaphragmatic breathing techniques. D) Initiate oxygenation therapy to increase saturation to 92%.

D, Initiate oxygenation therapy to increase saturation to 92%.

A nursing student wants to know why clients with chronic obstructive pulmonary disease tend to be polycythemic. What response by the nurse instructor is best? A) It is due to side effects of medications for bronchodilation. B) It is from overactive bone marrow in response to chronic disease. C) It combats the anemia caused by an increased metabolic rate. D) It compensates for tissue hypoxia caused by lung disease.

D, It compensates for tissue hypoxia caused by lung disease.

The nurse is caring for a client who is prescribed a long-acting beta2 agonist. The client states, The medication is too expensive to use every day. I only use my inhaler when I have an attack. How should the nurse respond? A) You are using the inhaler incorrectly. This medication should be taken daily. B) If you decrease environmental stimuli, it will be okay for you to use the inhaler only for asthma attacks. C) Tell me more about your fears related to feelings of breathlessness. D) It is important to use this type of inhaler every day. Lets identify potential community services to help you.

D, It is important to use this type of inhaler every day. Lets identify potential community services to help you.

.A client is on mechanical ventilation and the clients spouse wonders why ranitidine (Zantac) is needed since the client only has lung problems. What response by the nurse is best? A) It will increase the motility of the gastrointestinal tract. B) It will keep the gastrointestinal tract functioning normally. C) It will prepare the gastrointestinal tract for enteral feedings. D) It will prevent ulcers from the stress of mechanical ventilation.

D, It will prevent ulcers from the stress of mechanical ventilation.

The nurse has taught a client with cancer ways to prevent infection. What statement by the client indicates that more teaching is needed? A) I should take my temperature daily and when I don't feel well. B) I will wash my toothbrush in the dishwasher once a day. C) I wont let anyone share any of my personal items or dishes. D) Its alright for me to keep my pets and change the litter box.

D, Its alright for me to keep my pets and change the litter box.

A nurse cares for a client who has a pleural chest tube. Which action should the nurse take to ensure safe use of this equipment? A) Strip the tubing to minimize clot formation and ensure patency. B) Secure tubing junctions with clamps to prevent accidental disconnections. C) Connect the chest tube to wall suction at the level prescribed by the provider. D) Keep padded clamps at the bedside for use if the drainage system is interrupted.

D, Keep padded clamps at the bedside for use if the drainage system in interrupted.

A nurse cares for a client who has packing inserted for posterior nasal bleeding. Which action should the nurse take first? A) Assess the clients pain level. B) Keep the clients head elevated. C) Teach the client about the causes of nasal bleeding. D) Make sure the string is taped to the clients cheek.

D, Make sure the string is taped to the clients cheek.

A nurse assesses a client admitted with a brown recluse spider bite. Which priority assessment should the nurse perform to identify complications of this bite? A) Ask the client about pruritus at the bite site. B) Inspect the bite site for a bluish purple vesicle. C) Assess the extremity for redness and swelling. D) Monitor the clients temperature every 4 hours.

D, Monitor the client temperature every 4 hours.

A nurse is field-triaging clients after an industrial accident. Which client condition should the nurse triage with a red tag? A) Dislocated right hip and an open fracture of the right lower leg. B) Large contusion to the forehead and a bloody nose. C) Closed fracture of the right clavicle and arm numbness. D) Multiple fractured ribs and shortness of breath

D, Multiple fractured rids and shortness of breath.

An older client with peripheral vascular disease (PVD) is explaining the daily foot care regimen to the family practice clinic nurse. What statement by the client may indicate a barrier to proper foot care? A) I nearly always wear comfy sweatpants and house shoes. B) Im glad I get energy assistance so my house isn't so cold. C) My daughter makes sure I have plenty of lotion for my feet. D) My hands shake when I try to do things requiring coordination.

D, My hands shake when I try to do things requiring coordination.

A nurse assesses a clients respiratory status. Which information is of highest priority for the nurse to obtain? A) Average daily fluid intake. B) Neck circumference. C) Height and weight. D) Occupation and hobbies

D, Occupation and hobbies.

A student nurse is assessing the peripheral vascular system of an older adult. What action by the student would cause the faculty member to intervene? A) Assessing blood pressure in both upper extremities B) Auscultating the carotid arteries for any bruits C) Classifying capillary refill of 4 seconds as normal D) Palpating both carotid arteries at the same time

D, Palpating both carotid arteries at the same time.

A nurse assesses a client who has burn injuries and notes crackles in bilateral lung bases, a respiratory rate of 40 breaths/min, and a productive cough with blood-tinged sputum. Which action should the nurse take next? A) Administer furosemide (Lasix). B) Perform chest physiotherapy. C) Document and reassess in an hour. D) Place the client in an upright position.

D, Place the client in an upright position.

A hospitalized client has a platelet count of 58,000/mm3. What action by the nurse is best? A) Encourage high-protein foods. B) Institute neutropenic precautions. C) Limit visitors to healthy adults. D) Place the client on safety precautions.

D, Place the client on safety precautions.

A client is brought to the emergency department after sustaining injuries in a severe car crash. The clients chest wall does not appear to be moving normally with respirations, oxygen saturation is 82%, and the client is cyanotic. What action by the nurse is the priority? A) Administer oxygen and reassess. B) Auscultate the clients lung sounds. C) Facilitate a portable chest x-ray. D) Prepare to assist with intubation.

D, Prepare to assist with intubation.

A nurse assesses a client after administering a prescribed beta blocker. Which assessment should the nurse expect to find? A) Blood pressure increased from 98/42 mm Hg to 132/60 mm Hg B) Respiratory rate decreased from 25 breaths/min to 14 breaths/min C) Oxygen saturation increased from 88% to 96% D) Pulse decreased from 100 beats/min to 80 beats/min

D, Pulse decreased from 100 beats/min to 80 beats/min.

A new nurse has been assigned a client who is in the hospital to receive iodine-131 treatment. Which action by the nurse is best? A) Ensure the client is placed in protective isolation. B) Hand off a pregnant client to another nurse. C) No special action is necessary to care for this client. D) Read the policy on handling radioactive excreta.

D, Read the policy on handling radioactive excreta.

A nurse is caring for a client with acute pericarditis who reports substernal precordial pain that radiates to the left side of the neck. Which nonpharmacologic comfort measure should the nurse implement? A) Apply an ice pack to the clients chest. B) Provide a neck rub, especially on the left side. C) Allow the client to lie in bed with the lights down. D) Sit the client up with a pillow to lean forward on.

D, Sit the client up with a pillow to lean forward on.

A nurse prepares to provide perineal care to a client with meningococcal meningitis. Which personal protective equipment should the nurse wear? (Select all that apply.) A) Particulate respirator B) Isolation gown C) Shoe covers D) Surgical mask E) Gloves

D, Surgical mask. E, Gloves,

A nurse is caring for a client on mechanical ventilation. When double-checking the ventilator settings with the respiratory therapist, what should the nurse ensure as a priority? A) The client is able to initiate spontaneous breaths. B) The inspired oxygen has adequate humidification. C) The upper peak airway pressure limit alarm is off. D) The upper peak airway pressure limit alarm is on.

D, The upper peak airway pressure limit alarm is on.

nursing student is caring for a client with leukemia. The student asks why the client is still at risk for infection when the clients white blood cell count (WBC) is high. What response by the registered nurse is best? A) If the WBCs are high, there already is an infection present. B) The client is in a blast crisis and has too many WBCs. C) There must be a mistake; the WBCs should be very low. D) Those WBCs are abnormal and dont provide protection.

D, Those WBCs are abnormal and dont provide protection.

The nurse is reviewing the lipid panel of a male client who has atherosclerosis. Which finding is most concerning? A) Cholesterol: 126 mg/dL B) High-density lipoprotein cholesterol (HDL-C): 48 mg/dL C) Low-density lipoprotein cholesterol (LDL-C): 122 mg/dL D) Triglycerides: 198 mg/dL

D, Triglycerides: 198 mg/dL.

A nurse is caring for a client who is scheduled to undergo a thoracentesis. Which intervention should the nurse complete prior to the procedure? A) Measure oxygen saturation before and after a 12-minute walk. B) Verify that the client understands all possible complications. C) Explain the procedure in detail to the client and the family. D) Validate that informed consent has been given by the client.

D, Validate that informed consent has been given by the client.

The registered nurse assigns a client who has an open burn wound to a licensed practical nurse (LPN). Which instruction should the nurse provide to the LPN when assigning this client? A) Administer the prescribed tetanus toxoid vaccine. B) Assess the clients wounds for signs of infection. C) Encourage the client to breathe deeply every hour. D) Wash your hands on entering the clients room.

D, Wash your hands on entering th clients room.

A client with cancer is admitted to a short-term rehabilitation facility. The nurse prepares to administer the clients oral chemotherapy medications. What action by the nurse is most appropriate? A) Crush the medications if the client cannot swallow them. B) Give one medication at a time with a full glass of water. C) No special precautions are needed for these medications. D) Wear personal protective equipment when handling the medications.

D, Wear personal protective equipment when handling the medications.

A nurse teaches a client who has a history of heart failure. Which statement should the nurse include in this clients discharge teaching? A) Avoid drinking more than 3 quarts of liquids each day. B) Eat six small meals daily instead of three larger meals. C) When you feel short of breath, take an additional diuretic. D) Weigh yourself daily while wearing the same amount of clothing.

D, Weigh yourself daily while wearing the same amount of clothing.

A client has been admitted for suspected inhalation anthrax infection. What question by the nurse is most important? A) Are any family members also ill? B) Have you traveled recently? C) How long have you been ill? D) What is your occupation?

D, What is your occupation?

A nurse cares for a client who has a chest tube. When would this client be at highest risk for developing a pneumothorax? A) When the insertion site becomes red and warm to the touch. B) When the tube drainage decreases and becomes sanguineous. C) When the client experiences pain at the insertion site. D) When the tube becomes disconnected from the drainage system.

D, When the tube becomes disconnected from the drainage system.

A nurse assesses a client after a thoracentesis. Which assessment finding warrants immediate action? A) The client rates pain as a 5/10 at the site of the procedure. B) A small amount of drainage from the site is noted. C) Pulse oximetry is 93% on 2 liters of oxygen. D) The trachea is deviated toward the opposite side of the neck.

D, the trachea is deviated toward the opposite side of the neck.

What is the best tracker of fluid volume status from day to day?

Daily weight.

A nurse is reinforcing teaching with a client who has a new diagnosis of migraine headaches about interventions to reduce pain at the onset of a migraine. Which of the following instructions should the nurse include in the teaching?

Darken the lights.

A patient is being weaned from TPN and is expected to begin taking solid food today. The ongoing solution rate has been 100 ml/hr. The nurse anticipates that which prescription regarding the TPN solution will accompany the diet prescription?

Decrease the TPN rate to 50 mL/hr.

A nurse in a rehabilitation center is collecting data from a client who is recovering from a left hemisphere stroke. Which of the following findings should the nurse expect?

Difficulty with speech.

Nursing interventions for Digoxin?

Digoxin therapeutic level: 0.5 - 2.0. Toxicity can occur > 2.0; S/S: see/puke/poop green. Hypokalemia increases risk for toxicity. CHECK APICAL PULSE FOR 1 FULL MINUTE- If its < 60, HOLD IT!

A client had a percutaneous transluminal coronary angioplasty for peripheral arterial disease. What assessment finding by the nurse indicates a priority outcome for this client has been met?

Distal pulse on affected extremity 2+/4+.

A nurse teaches a client who is prescribed digoxin (Lanoxin) therapy. Which statement should the nurse include in this clients teaching?

Do not take this medication within 1 hour of taking an antacid.

A client is in the clinic a month after having a myocardial infarction. The client reports sleeping well since moving into the guest bedroom. What response by the nurse is best?

Do you have any concerns about sexuality?

Upon auscultation of a patient, the nurse hears normal S1 and S2. What is the most appropriate intervention by the nurse?

Document the findings.

A nurse cares for a client with burn injuries from a house fire. The client is not consistently oriented and reports a headache. Which action should the nurse take?

Draw blood for a carboxyhemoglobin level.

A nurse works at a community center for older adults. What self-management measure can the nurse teach the clients to prevent shock?

Drink fluids on a regular schedule.

A nurse is reinforcing teaching with a client who has a new diagnosis of primary open angle glaucoma (POAG). Which of the following information should the nurse include in the teaching?

Driving can be dangerous due to the loss of peripheral vision. Laser surgery can help reestablish the flow of aqueous humor.

A nurse is collecting data from a client who has a brain tumor. Which of the following findings indicates cranial nerve involvement?

Dysphagia.

A nurse is reinforcing teaching with the partner of a client who has a new diagnosis of Parkinson's disease about degenerative complications. The nurse should include in the teaching that which of the following manifestations is the priority?

Dysphagia.

A nurse assesses a client with mitral valve stenosis. What clinical manifestation should alert the nurse to the possibility that the clients stenosis has progressed?

Dyspnea on exertion.

What symptoms would the nurse expect to see in a client with Idiopathic thrombocytopenic purpura (ITP)?

Ecchymosis and petechiae due to low H and H.

A nurse is preparing to hang the first bag of TPN solution via a central line in the patient. The nurse should obtain which most essential piece of equipment before hanging the solution?

Electric infusion pump.

A nurse is reviewing the laboratory results of a lumbar puncture (LP) for a client who has manifestations of bacterial meningitis. The nurse should recognize which of the following findings is consistent with this diagnosis?

Elevated protein.

Primary prevention of cancer?

Eliminate cause. Change modifiable risk factors. Remove "at risk" cells. and vaccines.

A client has hemodynamic monitoring after a myocardial infarction. What safety precaution does the nurse implement for this client?

Ensure the balloon does not remain wedged.

A client arrives in the emergency department after being in a car crash with fatalities. The client has a nearly amputated leg that is bleeding profusely. What action by the nurse takes priority?

Ensure the client has a patent airway.

Where is S2 best heard?

Erbs point.

A home health care nurse is visiting an older client who lives alone after being discharged from the hospital after a coronary artery bypass graft. What finding in the home most causes the nurse to consider additional referrals?

Expired foods in the refrigerator.

A client with Hemophilia A is scheuled for surgery. What should the nurse expect to administer just prior to the operation.

Factor VIII.

What is Digoxin used for?

Fast HR (A-fib) for patient with heart failure.

What are Beta Blockers used for?

Fast HR- Atrial, SVT, and Ventricular (not first line for ventricular).

A nurse is reinforcing teaching with a class of new parents about otitis media. Which of the following manifestations should the nurse include in the teaching?

Feeling of fullness in the ear.

A client has clear fluid leaking from the nose following a basilar skull fracture. Which finding would alert the nurse that CSF is present?

Fluid separates into concentric rings and tests positive for glucose.

A nurse assesses a client with pericarditis. Which assessment finding should the nurse expect to find?

Friction rub at the left lower sternal border.

The nurse is caring for four hypertensive clients. Which druglaboratory value combination should the nurse report immediately to the health care provider?

Furosemide (Lasix)/potassium: 2.1 mEq/L/

A nurse assesses bilateral wheezes in a client with burn injuries inside the mouth. Four hours later the wheezing is no longer heard. Which action should the nurse take?

Gather appropriate equipment and prepare for an emergency airway.

A nurse teaches a client with heart failure about energy conservation. Which statement should the nurse include in this clients teaching?

Gather everything you need for a chore before you begin.

The client is noted to have a large amount of edema all throughout the body that does not indent. How should the nurse best document this finding?

Generalized non pitting edema.

How is sever folic acid deficiency anemia treated?

Give a folic acid injection.

A client has presented to the emergency department with an acute myocardial infarction (MI). What action by the nurse is best to meet The Joint Commissions Core Measures WWW.TESTBANKTANK.COM outcomes?

Give the client an aspirin.

What are some interventions for VTE?

Heparin, lovenox, or warfarin.

A client is in shock and the nurse prepares to administer insulin for a blood glucose reading of 208 mg/dL. The spouse asks why the client needs insulin as the client is not a diabetic. What response by the nurse is best?

High glucose. is common in shock and needs to be treated.

A client has peripheral arterial disease (PAD). What statement by the client indicates misunderstanding about self-management activities?

I can use a heating pad on my legs if its set on low.

A client is being discharged home after a large myocardial infarction and subsequent coronary artery bypass grafting surgery. The clients sternal wound has not yet healed. What statement by the client most indicates a higher risk of developing sepsis after discharge?

I hope I can get my water turned back on when I get home.

A nurse assesses a client in an outpatient clinic. Which statement alerts the nurse to the possibility of left-sided heart failure?

I must stop halfway up the stairs to catch my breath.

A nurse is reinforcing teaching with a client who is postoperative following cataract surgery and has an intraocular lens implant. Which of the following statements by the client indicates an understanding of the teaching?

I will avoid bending over.

After teaching a client who is being discharged home after mitral valve replacement surgery, the nurse assesses the clients understanding. Which client statement indicates a need for additional teaching?

I will have my teeth cleaned by my dentist in 2 weeks.

A nurse assesses a client who has a burn injury. Which statement indicates the client has a positive perspective of his or her appearance?

I with bathe and dress before breakfast.

The first line treatment for mild iron deficiency anemia is which of the following?

Increasing iron intake in foods contained. Oral supplements will be added if diet is not sufficient.

A nurse is reinforcing teaching with an adolescent client who has recurrent external otitis. Which of the following instructions should the nurse include in the teaching?

Instill a diluted alcohol solution into the ear after swimming.

After administering newly prescribed captopril (Capoten) to a client with heart failure, the nurse implements interventions to decrease complications. Which priority intervention should the nurse implement for this client?

Instruct the client to ask for assistance when rising from bed.

A nurse is reinforcing teaching with a client who is preoperative for cataract surgery. The nurse should include in the teaching that which of the following is an adverse effect of cataract surgery?

Intraocular hemorrhage.

3 types of monitoring for ICP?

Intraventricular catheter, subarachnoid screw, or epidural probe.

Why is MAP considered a better indicator of perfusion compared to SBP alone?

It accounts for the fact that two thirds of the cardiac cycle are spent in diastole.

A nurse prepares to administer intravenous cimetidine (Tagamet) to a client who has a new burn injury. The client asks, Why am I taking this medication? How should the nurse respond?

It helps prevent stomach ulcers, which are common after burns.

A student nurse asks what essential hypertension is. What response by the registered nurse is best?

It is HTN with no specific cause.

The provider requests the nurse start an infusion of an inotropic agent on a client. How does the nurse explain the action of these drugs to the client and spouse?

It slows the force of the hearts contractions.

A nurse is caring for several clients at risk for shock. Which laboratory value requires the nurse to communicate with the health care provider?

Lactate 6 mmol/L.

For a patient with trauma, what should the ICP be?

Less than 20 mmHg.

A nurse is collecting data from a client who is admitted to the facility for observation following a closed head injury. Which of the following data is the priority for the nurse to collect to detect a change in the client's neurologic status?

Level of consciousness.

What are examples of Na Channel Blockers?

Lidocaine and Mexiletine.

A nurse is reinforcing teaching with the family of a client who has stage 2 Alzheimer's disease (AD). Which of the following information should the nurse include in the teaching?

Limit choices offered to the client.

A nurse is assessing a client who had a myocardial infarction. Upon auscultating heart sounds, the nurse hears tan S3 heart sound. What action by the nurse is most appropriate?

Listen to the clients lungs.

Which of the following lab values would you expect for a client with pancytopenia?

Low platelets.

A student is caring for a client who suffered massive blood loss after trauma. How does the student correlate the blood loss with the clients mean arterial pressure (MAP)?

Lower blood volume lowers MAP.

A client in the cardiac stepdown unit reports severe, crushing chest pain accompanied by nausea and vomiting. What action by the nurse takes priority?

Maintain airway patency.

Causes of folic acid deficiency anemia include which of the following?

Malabsorption. Diet poor in green leafy veggies, liver, yeast, citrus fruits, and dried beans/nuts.

A student nurse needs to listen to the apical pulse for a full 60 seconds. The RN knows the nurse is listening in the correct place if the stethoscope is placed where?

Mitral valve.

Nursing interventions for K Channel Blockers?

Monitor HR and BP. Hold medication if SBP< 100 to HR <60.

Nursing interventions for Na Channel Blockers?

Monitor HR and BP. Hold medication if SBP< 100 to HR <60.

Nursing interventions for Ca Channel Blockers?

Monitor HR and BP. Hold medication if SBP< 100 to HR <60. Make sure patient does not consume grapefruit juice.

Nursing interventions for Selective Beta Blockers?

Monitor HR and BP. Hold medication if SBP< 100 to HR <60. No effect on lungs. Will mask s/s of hypoglycemia.

Nursing interventions for Nonselective Beta Blockers?

Monitor HR and BP. Hold medication if SBP< 100 to HR <60. Will also effect lungs and cause bronchoconstriction. Will mask s/s of hypoglycemia.

Nursing Interventions for Magnesium Sulfate?

Monitor magnesium level.

Nursing Interventions for Atropine Sulfate?

Monitor the HR before, during, and after.

A client has been diagnosed with hypertension but does not take the antihypertensive medications because of a lack of symptoms. What response by the nurse is best?

Most people with HTN do not have symptoms.

An older client with peripheral vascular disease (PVD) is explaining the daily foot care regimen to the family practice clinic nurse. What statement by the client may indicate a barrier to proper foot care?

My hands shake when I try to do things requiring coordination.

A nurse assesses a client admitted to the cardiac unit. Which statement by the client alerts the nurse to the possibility of right-sided heart failure?

My shoes fit really tight lately.

The nurse is assessing the motor function of an unconscious client. The nurse would plan to use which of the following to test the client's peripheral response to pain?

Nail bed pressure.

A client's blood pressure was 134/82 and is now currently 94/84. How would you describe the pulse pressure?

Narrowing

Aplastic anemia is typically treated with blood transfusions (dependent on cause) until the bone marrow begins working again. What type of fluid should be run with the PRBCs during infusion to prevent clumping/clotting of the blood?

Normal saline.

A client is receiving an infusion of alteplase (Activase) for an intra-arterial clot. The client begins to mumble and is disoriented. What action by the nurse takes priority?

Notifiy the Rapid response Team.

A nurse caring for a client notes the following assessments: white blood cell count 3800/ mm3, blood glucose level 198 mg/dL, and temperature 96.2 F (35.6 C). What action by the nurse takes priority?q

Notify the HCP immediately.

The nurse is caring for a client with a chest tube after a coronary artery bypass graft. The drainage slows significantly. What action by the nurse is most important?

Notify the provider immediately.

A nurse is preparing to hang fat emulsion (lipids) and notes that fat globules are visible at the top of the solution. The nurse should take which action?

Obtain a different bottle of solution.

A patient with TPN infusing has disconnected the tubing from the central line catheter. The nurse assesses the patient and suspects an air embolism. The nurse should immediately place the patient in which position?

On the left side, with the head lower than the feet.

A nurse in an acute care facility is preparing to admit a client who has myasthenia gravis. Which of the following supplies should the nurse place at the client's bedside?

Oral nasal suction equipment.

A nurse is caring for a client with a deep vein thrombosis (DVT). What nursing assessment indicates a priority outcome has been met?

Oxygen saturation of 98%.

What are generalized S/S associated with peripheral arterial problems?

Pain, decreased pulse, coolness, hair loss, dry skin, dusky skin, thickened toenails, ulcers, and muscle atrophy.

A student nurse is assessing the peripheral vascular system of an older adult. What action by the student would cause the faculty member to intervene?

Palpating both carotid arteries at the same time.

The priority problem with sickle cell crises is?

Perfusion and oxygenation of tissues.

A nurse assesses a client who has burn injuries and notes crackles in bilateral lung bases, a respiratory rate of 40 breaths/min, and a productive cough with blood-tinged sputum. Which action should the nurse take next?

Place the client in an upright position.

A client with heart failure is being assessed for fluid volume overload. The nurse knows that JVD should be assess by completing what?

Place the client supine with the HOB at 30-45 degrees and turn the head away.

A client had an acute myocardial infarction. What assessment finding indicates to the nurse that a significant complication has occurred?

Poor peripheral pulses and cool skin.

A client undergoing hemodynamic monitoring after a myocardial infarction has a right atrial pressure of 0.5 mm Hg. What action by the nurse is most appropriate?

Prepare to administer a fluid bolus.

What are examples of Beta Blockers?

Propanolol, Metoprolol, and Sotalol.

A nurse is reinforcing discharge teaching with the family of a client who has a new diagnosis of a seizure disorder. The nurse should instruct the client's family to take which of the following actions first in the event of a seizure?

Protect the client's head.

Nursing intervention for Adenosine?

Rapid IVP follow by slammed flush. Have crash cart on hand. Monitor continuous EKG.

A nurse is collecting data from a client who has a high thoracic spinal cord injury. The nurse should identify which of the following findings as a manifestation of autonomic dysreflexia?

Report of a headache.

A client in shock has been started on dopamine. What assessment finding requires the nurse to communicate with the provider immediately?

Report of chest heaviness.

A nurse is reviewing the record of a child with increased ICP and notes that the child has exhibited signs of decerebrateposturing. On assessment of the child, the nurse would expect to note which of the following if this type of posturing was present?

Rigid extension and pronation of the arms and legs.

A nurse is caring for a client who has a closed head injury. The nurse should place the client in which of the following positions?

Semi-Fowler's.

A nurse reviews the laboratory results for a client who was burned 24 hours ago. Which laboratory result should the nurse report to the health care provider immediately?

Serum potassium: 6.5 mEq/L.

A nurse is collecting data from a client who has a new diagnosis of acute angle closure glaucoma. The nurse should anticipate the client to report which of the following manifestations?

Severe eye pain.

The client is having a lumbar puncture performed. The nurse would plan to place the client in which position for the procedure?

Side-lying, with legs pulled up and head bent down onto the chest.

A nurse is caring for a client with acute pericarditis who reports substernal precordial pain that radiates to the left side of the neck. Which nonpharmacologic comfort measure should the nurse implement?

Sit the client up with a pillow to lean forward on.

The nurse teaches burn prevention to a community group. Which statement by a member of the group should cause the nurse the greatest concern?

Sometimes I wake up at night and smoke.

A nurse is collecting data from a client who has a recent head trauma and a urine output of 600 mL/hr. The nurse suspects the client has manifestations of diabetes insipidus (DI). Which of the following laboratory values should the nurse plan to obtain to monitor for DI?

Specific gravity.

A nurse cares for a client with infective endocarditis. Which infection control precautions should the nurse use?

Standard Precautions.

A patient has mild hemolytic anemia. What therapy does the nurse expect to administer to this client?

Steroids can be used for mild hemolytic therapy.

A client is seen in the HCP office for management of Thrombotic Thrombocytopenic purpura (TTP). Which of the following prescriptions does the nurse expect for this client?

Stop platelet aggregation by using immunosuppressants and anti platelets, such as aspirin.

A client is receiving an infusion of tissue plasminogen activator (t-PA). The nurse assesses the client to be disoriented to person, place, and time. What action by the nurse is best?

Stop the infusion and call the provider.

A nurse is collecting data from a client who has a new diagnosis of mastoiditis. Which of the following manifestations should the nurse expect?

Swelling behind the affected ear.

What are generalized S/S associated with peripheral venous problems?

Swelling, warmth, red, static dermatitis if stasis present, and stasis ulcers.

A nurse is preparing to change the TPN solution bag and tubing. The patient's central line is located in the right subclavian vein. The nurse asks the patient to take which essential action during the tubing change?

Take a deep breath, hold it, and bear down.

When caring for a client with head trauma, the nurse notes a small amount of clear, watery fluid oozing from the client's nose. What should the nurse do?

Test the drainage for glucose.

A client received tissue plasminogen activator (t-PA) after a myocardial infarction and now is on an intravenous infusion of heparin. The clients spouse asks why the client needs this medication. What response by the nurse is best?

The Heparin keeps the artery from getting blocked again.

A client with coronary artery disease (CAD) asks the nurse about taking fish oil supplements. What response by the nurse is best?

The best source is fish, but pulls have benefits too.

A nurse reviewing the medical history of a client who is scheduled for a magnetic resonance imaging (MRI) examination of the cervical vertebra. The nurse should alert the provider to which of the following information in the client's history that is a contraindication to the procedure?

The client has a pacemaker.

A client admitted to the hospital with a subarachnoid hemorrhage has complaints of severe headache, nuchal rigidity, and projectile vomiting. The nurse knows lumbar puncture (LP) would be contraindicated in this client in which of the following circumstances?

The client needs mechanical ventilation.

A nurse is caring for a client who has a traumatic brain injury and assumes a decerebrate posture in response to noxious stimuli. Which of the following reactions should the nurse anticipate when drawing a blood sample?

The client rigidly extends his arms.

A nurse is collecting data from a client who has a closed head injury and is receiving mannitol for manifestations of increased intracranial pressure (ICP). Which of the following findings indicates to the nurse that the medication is having a therapeutic effect?

The client's urine output is 250 mL/hr.

A nurse assesses a client admitted with deep partial-thickness and full- thickness burns on the face, arms, and chest. Which asse

Urine output of 20 mL/hr.

What is Magnesium Sulfate used for?

Ventricular dysrhythmias that are due to magnesium deficiency.

What are K Channel Blockers used for?

Ventricular dysrhythmias.

What are Na Channel Blockers used for?

Ventricular dysrhythmias.

What are examples of Ca Channel Blockers?

Verapamil and Diltiazem.

A nurse if reinforcing teaching with the family of a client who has a new diagnosis of amyotrophic lateral sclerosis (ALS). The nurse should include in the teaching that which of the following findings is an early manifestation of ALS?

Weakness of the distal extremities.

A nurse is collecting data from a client who has Guillain-Barre syndrome. Which of the following findings should the nurse expect?

Weakness of the lower extremities.

A nurse monitors a patient receiving TPN for complications of the therapy and should assess the patient for which manifestation of hyperglycemia?

Weakness, thirst, and increased urine output.

A nurse teaches a client who has a history of heart failure. Which statement should the nurse include in this clients discharge teaching?

Weigh yourself daily while wearing the same amount of clothing.

A nurse cares for a client with right-sided heart failure. The client asks, Why do I need to weigh myself every day? How should the nurse respond?

Weight is the best indication that you are gaining or losing fluid.

Thrombocytopenia(decreased platelets)?

When Platelet count less than 50,000, minor trauma can result in prolonged bleeding; Less than 20,000 can result in spontaneous/uncontrolled bleeding (monitor H&H for bleeding too) Observe IV sites for bleeding/bruising; Look for petechiae, ecchymoses, bleeding gums/nose, hematuria 1. Initiate Bleeding Precautions: Provide safe environment; Handle patient gently; Avoid injections, Apply firm pressure for extended time when injections or venipunctures are necessary and use ice if needed; Electric razor & Soft bristled tooth brush; avoid contact sports/activities; avoid blowing nose; no aspirin or NSAIDS 2. Administer growth factors to stimulate marrow production if appropriate (would not be appropriate for Leukemia) (Neumega) 3. Administer Platelets if needed 4. Transfuse PRBCs if needed/bleeding present

A nurse cares for a client who has burn injuries. The clients wife asks, When will his high risk for infection decrease? How should the nurse respond?

When all of his un wounds have closed.

A nurse is collecting data from a client following a recent head injury. Which of the following findings should the nurse recognize as a manifestation of increased intracranial pressure?

Widened pulse pressure.

A nurse cares for a client with end-stage heart failure who is awaiting a transplant. The client appears depressed and states, I know a transplant is my last chance, but I dont want to become a vegetable. How should the nurse respond?

Would you like information about advance directives?

A nurse cares for an older adult client with heart failure. The client states, I dont know what to do. I dont want to be a burden to my daughter, but I cant do it alone. Maybe I should die. How should the nurse respond?

Would you like to talk more about this?

A nurse is reinforcing discharge teaching with a client who is postoperative following scleral buckling to repair a detached retina. Which of the following instructions should the nurse include in the teaching?

You should expect to see flashing lights in front of the affected eye after the procedure.

A nurse cares for a client who has facial burns. The client asks, Will I ever look the same? How should the nurse respond?

You will not look exactly the same but cosmetic surgery will help.

After teaching a client who is recovering from a heart transplant to change positions slowly, the client asks, Why is this important? How should the nurse respond?

Your new heart is not connected to the nervous system and is unable to respond to decreases in blood pressure caused by position changes.


Ensembles d'études connexes

Entrepreneurship Exam 3 (Chapters 10-14)

View Set

History Exam: Ch. 6- Scientific Revolution and the Enlightenment The statements below best describe which period? * Captionless Image

View Set

Chapter 7 Exam: Group Life Insurance

View Set

A&P Muscular System Lecture EXAM #3

View Set

HRMT 101 Week 2 - Basic Principles of APA Style & Formatting

View Set

NU141- Chapter 55 Drugs Acting on the Lower Respiratory Tract

View Set

Pharmacology ATI Oral Medications

View Set

IT 467 midterm - SQL and firewall zone policy

View Set

CH9: Disability Income and Related Insurance

View Set